ML071560216

From kanterella
Jump to navigation Jump to search
11-2005-Draft Written Exam Question Worksheet
ML071560216
Person / Time
Site: South Texas  STP Nuclear Operating Company icon.png
Issue date: 05/18/2005
From:
NRC Region 4
To:
References
Download: ML071560216 (176)


Text

ES-401 Written Examination Question Worksheet Form ES-401-5 NUREG-1021 Revision-9 Examination Outline Cross-reference: Level RO SRO Tier # 1 Group # 1 K/A # 007.EK2.03 Importance Rating 3.8 Proposed Question 1:

Unit 1 was operating at 100% power when a reactor trip occurred. Plant conditions immediately after the trip:

  • Train R Rx Trip Breaker is OPEN
  • Train S Rx Trip Breaker is CLOSED
  • SUR is slightly negative Based on these indications, the NEXT appropriate operator action would be to:

A. Transition to 0POP05-EO-FRS1, Response to Nuclear Power Generation - ATWS B. Verify the turbine is tripped C. Manually insert control rods D. Commence Emergency Boration Proposed Answer: B Explanation (Optional):

Immediate actions of 0POP05-EO-EO00 must be completed prior to transitioning to another procedure or action, thus Answer B is correct.

Technical

Reference:

0POP05-EO-EO00 (Rev 17)

Proposed references to be provided to applicants during examination:

None Learning Objective: (As available)

Question Source: Bank #

Modified Bank # (Note changes or attach parent)

New X Question History: Last NRC Exam Cognitive Level: Memory or Fundamental Knowledge Comprehension or Analysis X 10 CFR Part 55 Content: 55.41 10 55.43 Comments:

KA: 007 Reactor Trip, EK2.03 : Knowledge of the interrelations between a reactor trip and the following - Reactor trip status panel

ES-401 Written Examination Question Worksheet Form ES-401-5 NUREG-1021 Revision-9 Examination Outline Cross-reference: Level RO SRO Tier # 1 Group # 1 K/A # 008.AA1.04 Importance Rating 2.8*

Proposed Question 2:

During a loss of reactor coolant accident, AFW Pump 11 failed to start. According to 0POP05-EO-EO10, Loss of Reactor or Secondary Coolant, which ONE of the following describes the required operator actions?

A. Locally start AFW Pump 11 in order to feed SG A B. Manually start MFW Pump 11 in order to feed SG A C. Isolate SG A while maintaining AFW flow to SG's B, C, and D D. Cross connect operating AFW pumps to feed SG A Proposed Answer: D Explanation (Optional):

0POP05-EO-EO10, Step 8 states IF any AFW fails to start, THEN a) reset all SG LO-LO Level AFW actuations, b) close applicable AFW Reg Valve, c) open applicable AFW cross connects, d) control AFW flow to < 675 gpm per AFW pump.

Technical

Reference:

0POP05-EO-EO10 (Rev 17, page 8 of 26)

Proposed references to be provided to applicants during examination:

None Learning Objective: (As available)

Question Source: Bank #

Modified Bank # (Note changes or attach parent)

New X Question History: Last NRC Exam Cognitive Level: Memory or Fundamental Knowledge X Comprehension or Analysis 10 CFR Part 55 Content: 55.41 10 55.43 Comments:

KA: 008 PZR Vapor Space Accident, AA1.04 - Ability to operate and/or monitor the following as they apply to the PZR Vapor Space Accident: Feedwater pumps

ES-401 Written Examination Question Worksheet Form ES-401-5 NUREG-1021 Revision-9 Examination Outline Cross-reference: Level RO SRO Tier # 1 Group # 1 K/A # 009.G2.1.25 Importance Rating 2.8 Proposed Question 3:

A SBLOCA has occurred on Unit 1. Operators have transitioned to 0POP05-EO-ES12, Post LOCA Cooldown and Depressurization, and are currently at Step 5 to determine required boron concentration to establish adequate Shutdown Margin per Plant Curve Book Figure 5.5, 68 °F curve.

Plant conditions:

  • RCS pressure is 450 psig and slowing rising
  • Highest core exit thermocouple is 450 °F
  • Cycle Burnup is 12,000 MWD/MTU Using the attached figure, what is the minimum boron concentration (ppm) the RCS should be borated to ensure adequate Shutdown Margin for this step?

A. 1068 ppm B. 1207 ppm C. 1259 ppm D. 1404 ppm Proposed Answer: D Explanation (Optional):

A. Incorrect - this number corresponds to the 500 °F curve, one column to the right of 450 °F B. Incorrect - this number corresponds to the 450 °F curve C. Incorrect - this number corresponds to the 400 °F curve, one column to the left of 450 °F D. Correct - this number corresponds to the 68 °F curve at 12,000 MWD/MTU 0POP05-EO-ES12 (Rev 11); Unit 1 Plant Curve Book, Figure 5.5 Technical

Reference:

(Unit 1, Cycle 13)

Proposed references to be provided to applicants during examination:

Unit 1 Plant Curve Book Figure 5.5 (Shutdown Margin Limit Curve)

Learning Objective: (As available)

Question Source: Bank #

Modified Bank # (Note changes or attach parent)

New X Question History: Last NRC Exam Cognitive Level: Memory or Fundamental Knowledge

Comprehension or Analysis X 10 CFR Part 55 Content: 55.41 10 55.43 Comments:

KA: 009 Small Break LOCA, G2.1.25: Ability to obtain and interpret station reference materials such as graphs, monographs, and tables which contain performance data.

ES-401 Written Examination Question Worksheet Form ES-401-5 NUREG-1021 Revision-9 Examination Outline Cross-reference: Level RO SRO Tier # 1 Group # 1 K/A # 011.EA2.08 Importance Rating 3.4*

Proposed Question 4:

Which ONE of the following describes the post-LOCA condition that must be met to implement 0POP05-EO-ES13, Transfer to Cold Leg Recirculation?

A. RCS pressure < 415 psig with LHSI flow > 500 gpm B. Containment Wide Range Water Level > 59 C. RWST level less than 14% (75,000 gal.)

D. Two trains of HHSI/LHSI verified capable of Cold Leg Recirculation Proposed Answer: C Explanation (Optional):

A. Incorrect - Step 21 checks RCS pressure to determine if RCS cooldown and depressurization is required.

B. Incorrect - containment water level not an entry condition to ES13.

C. Correct - Step 22 states that if this condition is met, then go to ES13 D. Incorrect - Step 20 states that one train of EITHER HHSI OR LHSI must be verified available but is not used as a transition criteria Technical

Reference:

0POP05-EO-EO10, Loss of Reactor or Secondary Coolant (Rev 17)

Proposed references to be provided to applicants during examination:

None Learning Objective: (As available)

Question Source: Bank #

Modified Bank # (Note changes or attach parent)

New X Question History: Last NRC Exam Cognitive Level: Memory or Fundamental Knowledge X Comprehension or Analysis 10 CFR Part 55 Content: 55.41 10 55.43 Comments:

KA: 011 Large Break LOCA, EA2.08 - Ability to determine or interpret the following as they apply to a Large Break LOCA: Conditions necessary for recovery when accident reaches stable phase

ES-401 Written Examination Question Worksheet Form ES-401-5 NUREG-1021 Revision-9 Examination Outline Cross-reference: Level RO SRO Tier # 1 Group # 1 K/A # 015/017.AA2.08 Importance Rating 3.4 Proposed Question 5:

Unit 1 is operating Mode 1 at 100% power when an ICS alarm is received. The operator determines there is an alarm on RCP 1B Motor Upper Thrust Bearing temperature.

In accordance with 0POP04-RC-0002, Reactor Coolant Pump Off Normal, what is the next prescribed action regarding this condition?

A. Trip the Reactor B. Validate the alarm by checking RCP 1B motor amps locally C. Stop RCP 1B D. Commence a rapid load reduction Proposed Answer: B Explanation (Optional):

0POP04-RC-002, Step 2.0 RNO has the operator validate RCP high thrust bearing temperature prior to taking action. The intent is to always validate the exceeded parameter.

A. Incorrect - Would be correct if alarm was for RCP radial bearing temp exceeded setpoint B. Correct - Operator must validate parameter prior to taking action C. Incorrect - plausible distractor for RCP alarm D. Incorrect - Plausible distractor if candidate anticipates a reactor trip due to alarm 0POP04-RC-0002,Reactor Coolant Pump Off Normal (Rev 22, Technical

Reference:

pages 6 and 41)

Proposed references to be provided to applicants during examination:

None Learning Objective: (As available)

Question Source: Bank #

Modified Bank # (Note changes or attach parent)

New X Question History: Last NRC Exam Cognitive Level: Memory or Fundamental Knowledge X Comprehension or Analysis 10 CFR Part 55 Content: 55.41 10 55.43 Comments:

KA: 015/017 RCP Malfunctions, AA2.08 - Ability to determine and interpret the following as

they apply to the RCP Malfunctions (Loss of RC Flow): When to secure RCPs on high bearing temperature

ES-401 Written Examination Question Worksheet Form ES-401-5 NUREG-1021 Revision-9 Examination Outline Cross-reference: Level RO SRO Tier # 1 Group # 1 K/A # 022.AK3.03 Importance Rating 3.1*

Proposed Question 6:

When placing Excess Letdown in service and flushing to the RCDT, why should caution be observed when establishing excess letdown flow?

A. To prevent damage to the CCW side of the Excess Letdown Heat Exchanger and loss of reactor coolant through the Seal Leakoff Return Header relief valve.

B. To prevent damage to the Seal Water Return Filter caused from overpressure.

C. To prevent lifting the RCDT pressure relief valve.

D. To prevent cavitating the RCDT pump(s).

Proposed Answer: C Explanation (Optional):

The procedure used to place Excess Letdown in service contains a CAUTION that states the RCDT pressure relief valve may lift when flushing the line. Thus, C is the correct answer.

Technical

Reference:

0POP02-CV-0004 CVCS Subsystem (Rev. 37, Section 13)

Proposed references to be provided to applicants during examination:

None Learning Objective: (As available)

Question Source: Bank #

Modified Bank # (Note changes or attach parent)

New X Question History: Last NRC Exam Cognitive Level: Memory or Fundamental Knowledge X Comprehension or Analysis 10 CFR Part 55 Content: 55.41 10 55.43 Comments:

KA: 022 Loss of Reactor Coolant Makeup, AK3.03 - Knowledge of the reasons for the following responses as they apply to the Loss of Reactor Coolant Makeup: Performance of lineup to establish excess letdown after determining need.

ES-401 Written Examination Question Worksheet Form ES-401-5 NUREG-1021 Revision-9 Examination Outline Cross-reference: Level RO SRO Tier # 1 Group # 1 K/A # 026.AA1.06 Importance Rating 2.9 Proposed Question 7:

Unit 2 is operating in Mode 1 with the following CCW pump lineup:

  • Pump 2A - running
  • Pump 2B - standby
  • Pump 2C - tagged out for maintenance A failure of Pump 2A discharge valve causes it to drift closed. Header pressure drops to 80 psig. Which ONE of the following describes the system response an operator would expect on normal letdown? (Assume normal letdown is in service.)

A. Letdown flow diverts to the Recycle Holdup Tank (RHUT)

B. Initial rise then drop in letdown temperature downstream of the Letdown Heat Exchanger C. Initial rise then drop in letdown temperature downstream of the Seal Water Heat Exchanger D. Letdown flow diverts to the RCDT Proposed Answer: B Explanation (Optional):

As Pump 2A discharge valve closes, discharge header pressure will drop causing reduced CCW flow to the letdown system components cooled by CCW. Since discharge header pressure does not reach the setpoint to start the standby pump (76 psig), the lower flow condition will remain.

A - Incorrect. This would occur if VCT level were to get too high.

B - Correct. Lowering CCW flow will cause letdown temp to rise, then as TV-4494 opens, the temperature will lower C - Incorrect. CCW flow to the Seal Water Heat Exchanger is not controlled by a temp controlled valve so temp will rise and remain at the higher temperature.

D - Incorrect - Excess letdown can be diverted to the RCDT but not normal letdown.

LOT201.06.HO.01 Rev. 12 (CVCS);

Technical

Reference:

LOT201.12.HO.01 Rev. 11 (CCW);

P&ID 5R209F05020 Sh 1 (CCW)

Proposed references to be provided to applicants during examination:

None Learning Objective: (As available)

Question Source: Bank #

Modified Bank # (Note changes or attach parent)

New X Question History: Last NRC Exam Cognitive Level: Memory or Fundamental Knowledge Comprehension or Analysis X 10 CFR Part 55 Content: 55.41 7 55.43 Comments:

KA: 026 Loss of Component Cooling Water, AA1.06 - Ability to operate and/or monitor the following as they apply to the Loss of Component Cooling Water: Control of flow rates to components cooled by the CCWS

ES-401 Written Examination Question Worksheet Form ES-401-5 NUREG-1021 Revision-9 Examination Outline Cross-reference: Level RO SRO Tier # 1 Group # 1 K/A # 027.G2.2.24 Importance Rating 2.6 Proposed Question 8:

Unit 1 is operating at 100% power. Maintenance has requested to tag out Group A and B Backup Heaters in order to investigate a spurious ground. Which ONE of the following describes the operator's primary concern with this request?

A. An LCO would be entered since all Backup Heater Groups are required to be operable.

B. An LCO would be entered if, during maintenance, any of the remaining Backup Heater groups are declared inoperable.

C. An LCO would be entered if, during maintenance, the Control Group of heaters is declared inoperable.

D. An LCO would be entered since two of three ESF powered pressurizer heater groups are required to be operable.

Proposed Answer: D Explanation (Optional):

Question tests an LCO entry condition.

A. Incorrect - Tech Spec 3.4.3 states "at least two groups of pressurizer heaters supplied by ESF power each having a capacity of at least 175 kW" not all Backup Heater Groups B. Incorrect - An LCO would be entered as soon as Groups A and B were inop C. Incorrect - An LCO would be entered as soon as Groups A and B were inop D. Correct - An LCO is entered as soon as Groups A and B are inop Technical

Reference:

LOT20104.HO.01 Rev. 7 (PAGE 3 OF 11); Tech Spec 3.4.3 Proposed references to be provided to applicants during examination:

None Learning Objective: (As available)

Question Source: Bank #

Modified Bank # (Note changes or attach parent)

New X Question History: Last NRC Exam Cognitive Level: Memory or Fundamental Knowledge Comprehension or Analysis X 10 CFR Part 55 Content: 55.41 5 55.43 Comments:

KA: 027 Pressurizer Pressure Control Malfunction, G2.2.24 - Ability to analyze the affect of

maintenance activities on LCO status ES-401 Written Examination Question Worksheet Form ES-401-5 NUREG-1021 Revision-9 Examination Outline Cross-reference: Level RO SRO Tier # 1 Group # 1 K/A # 029.EK1.03 Importance Rating 3.6 Proposed Question 9:

A plant transient has occurred. The crew is performing 0POP05-EO-FRS1, Response to Nuclear Power Generation - ATWS.

Complete the following statement: Negative reactivity will be added to the reactor MORE RAPIDLY if the operator _________________.

A. bypasses the letdown demineralizer.

B. closes one of the RMW non-essential header isolation valves.

C. increases the boric acid flow controller setpoint.

D. decreases the boric acid flow controller setpoint.

Proposed Answer: C Explanation (Optional):

Answer A is incorrect because bypassing the demineralizer will have no effect on the rate of boron addition or deletion to the reactor. Answer B is incorrect because closing one of the isolation valves will reduce the chance of a boron dilution event. Answer D is incorrect because decreasing the setpoint will reduce the rate of boron addition to the reactor. Answer C is correct because this will increase the rate of boron addition to the reactor thereby adding negative reactivity at a faster rate.

Technical

Reference:

0POP05-E0-FRS1 Proposed references to be provided to applicants during examination:

None Learning Objective: (As available)

Question Source: Bank #

Modified Bank # (Note changes or attach parent)

New X Question History: Last NRC Exam Cognitive Level: Memory or Fundamental Knowledge Comprehension or Analysis X 10 CFR Part 55 Content: 55.41 5 55.43 Comments:

KA: 029 ATWS, EK1.03 - Knowledge of the operational implications of the following concepts as they apply to ATWS: Effects of boron on reactivity.

ES-401 Written Examination Question Worksheet Form ES-401-5 NUREG-1021 Revision-9 Examination Outline Cross-reference: Level RO SRO Tier # 1 Group # 1 K/A # 038.EK1.02 Importance Rating 3.2 Proposed Question 10:

Given the following conditions for Unit 1:

  • RCS Subcooling is 75 °F
  • PZR Pressure = 1600 psig
  • SG A pressure = 1020 psig and slowly rising
  • SG B , C, D pressures = 675 psig What action should now be performed to minimize leakage flow from the RCS to the ruptured SG?

A. Open one PZR PORV B. Increase feedwater flow to SG A C. Initiate normal PZR spray D. Lower the Steam Dump no-load reference pressure setpoint Proposed Answer: C Explanation (Optional):

EO30, Step 19 directs the operator to depressurize the RCS to minimize break flow and refill the PZR using normal pressurizer spray if available. If not available, then use Aux Spray. If Aux Spray not available, then use one PZR PORV. The given conditions include RCPs are running thus normal PZR spray is available.

Technical

Reference:

0POP05-EO-EO30 (Rev 16, page 20 of 40)

Proposed references to be provided to applicants during examination:

None Learning Objective: (As available)

Question Source: Bank # INPO Modified Bank # (Note changes or attach parent)

New Question History: Last NRC Exam Cognitive Level: Memory or Fundamental Knowledge Comprehension or Analysis X

10 CFR Part 55 Content: 55.41 10 55.43 Comments:

KA: 038 Steam Generator Tube Rupture, EK1.02 - Knowledge of the operational implications of the following concepts as they apply to SGTR: Leak rate vs. pressure drop One distractor changed to reflect STP procedure guidance and make it more plausible (replaced 'Terminate SI and stop SI pumps' with 'Open one PZR PORV')

ES-401 Written Examination Question Worksheet Form ES-401-5 NUREG-1021 Revision-9 Examination Outline Cross-reference: Level RO SRO Tier # 1 Group # 1 K/A # 040.AA1.10 Importance Rating 4.1 Proposed Question 11:

While Unit 1 was operating at 100% power, a main steam line rupture occurred upstream of the Main Steam Isolation Valve for SG 1D. After completing the applicable steps of 0POP05-EO-EO00, the Shift Supervisor announced transition to 0POP05-EO-EO20.

What operator actions are required to isolate AFW to the ruptured SG per 0POP05-EO-EO20?

A. Reset SI; reset ESF load sequencers; reset SG LO-LO level AFW actuations; trip turbine driven AFW pump, verify automatic closure of AFW OCIV for SG 1D B. Reset SI; reset ESF load sequencers; reset SG LO-LO level AFW actuations; trip turbine driven AFW pump; manually close AFW OCIV for SG 1D C. Verify FWIV, FWIB, FW Preheater Bypass, FRV and LPFRV valves for SG 1D closed D. Close FWIV, FWIB, FW Preheater Bypass, FRV and LPFRV valves for SG 1D Proposed Answer: B Explanation (Optional):

A. Incorrect - AFW OCIV for SG 1D does not close automatically B. Corrrect - Step 4 of Tech Ref identifies five actions to isolate AFW to affected SG C. Incorrect - These steps are required to isolate Main FW to affected SG D. Incorrect - Identified valves are for Main FW, and these are closed automatically Technical

Reference:

0POP05-EO-EO20 (Rev 8, Step 4b)

Proposed references to be provided to applicants during examination:

None Learning Objective: (As available)

Question Source: Bank #

Modified Bank # (Note changes or attach parent)

New X Question History: Last NRC Exam Cognitive Level: Memory or Fundamental Knowledge X Comprehension or Analysis 10 CFR Part 55 Content: 55.41 10 55.43

Comments:

KA: 040 Steam Line Rupture, AA1.10 - Ability to operate and/or monitor the following as they apply to Steam Line Rupture: AFW system

ES-401 Written Examination Question Worksheet Form ES-401-5 NUREG-1021 Revision-9 Examination Outline Cross-reference: Level RO SRO Tier # 1 Group # 1 K/A # 054.AA1.04 Importance Rating 4.4 Proposed Question 12:

The control room operators are responding to a RED condition on the heat sink status tree. While they attempt to restore feed flow to a steam generator, conditions degrade to the point that RCS bleed-and-feed must be established.

The reason RCS bleed-and-feed must be established QUICKLY is to prevent which ONE of the following:

A. An overpressurization challenge to the reactor vessel.

B. The inability to provide sufficient injection flow for core cooling due to high RCS pressure C. High temperature and pressure failure of Steam Generator tubes D. A rapid RCS overpressurization, followed by a rapid RCS depressurization due to RCP seal failures Proposed Answer: B Explanation (Optional):

0POP05-EO-FRH1, Response to Loss of Secondary Heat Sink, contains a CAUTION statement prior to step 10 that states 'Steps 10-13 shall be performed quickly in order to establish RCS heat removal by RCS bleed and feed.' LOT504.33 Rev. 9 states the basis for this statement as 'Once secondary heat sink has degraded, RCS bleed and feed must be established within several minutes to prevent or minimize core uncovery.'

0POP05-EO-F003, Heat Sink Critical Safety Function Status Tree (Rev 5, page 2 of 2);

Technical

Reference:

0POP05-EO-FRH1, Response to Loss of Secondary Heat Sink (Rev 14, page 7 of 22);

LOT504.33 (Rev 9, page 11 of 19)

Proposed references to be provided to applicants during examination:

None Learning Objective: (As available)

Question Source: Bank # INPO Modified Bank # (Note changes or attach parent)

New Question History: Last NRC Exam Cognitive Level: Memory or Fundamental Knowledge X Comprehension or Analysis

10 CFR Part 55 Content: 55.41 10 55.43 Comments:

KA: 054 Loss of Main Feedwater, AA1.04 - Ability to operate and/or monitor the following as they apply to Loss of Main Feedwater INPO bank question - Braidwood 1, dated 7/17/2002

ES-401 Written Examination Question Worksheet Form ES-401-5 NUREG-1021 Revision-9 Examination Outline Cross-reference: Level RO SRO Tier # 1 Group # 1 K/A # 055.EK3.02 Importance Rating 4.3 Proposed Question 13:

Unit 1 has experienced a Loss of All AC Power. The crew is implementing 0POP05-EO-EC00, Loss of All AC Power that requires certain actions be accomplished prior to transitioning to the next applicable EOP.

Which one of the following actions is NOT required prior to transitioning from 0POP05-EO-EC00?

A. Power restored to any AC ESF bus B. Pressurizer PORV's closed C. RCP seal injection flow established or isolated D. SG's isolated except AFW Proposed Answer: D Explanation (Optional):

CIP states "When power is restored to any AC ESF bus, AND Steps 1 through 7 have been completed, THEN start recovery actions with Step 25."

A. Incorrect - Power to any AC ESF must occur prior to recovery B. Incorrect - PZR PORVs are closed in Step 4 C. Incorrect - RCP seal injection flow actions are identified in Step 3 D. Correct - Isolating SG's (except AFW) is performed in Step 10, after transitioning to procedure and step in affect, Step 7 Technical

Reference:

0POP05-EO-EC00 (Rev 16)

Proposed references to be provided to applicants during examination:

None Learning Objective: (As available)

Question Source: Bank #

Modified Bank # (Note changes or attach parent)

New X Question History: Last NRC Exam Cognitive Level: Memory or Fundamental Knowledge X Comprehension or Analysis 10 CFR Part 55 Content: 55.41 10 55.43 Comments:

Replaced original KA (EK3.01) - answer requires TS bases knowledge that is N/A to ROs KA: 055 Loss of Offsite and Onsite Power (Station Blackout), EK3.02 - Knowledge of the reasons for the following responses as they apply to Station Blackout: Actions contained in EOP for loss of offsite and onsite power

ES-401 Written Examination Question Worksheet Form ES-401-5 NUREG-1021 Revision-9 Examination Outline Cross-reference: Level RO SRO Tier # 1 Group # 1 K/A # 056.AK1.03 Importance Rating 3.1*

Proposed Question 14:

While operating at 100% power, Unit 1 experienced a reactor trip due to a loss of offsite power. The following plant conditions exist:

  • Tavg is 531 °F
  • Tcold is at 527 °F
  • Thot is at 534 °F
  • Average of the five (5) hottest CETs is 538 °F
  • Pressurizer pressure is at 2185 psig Which ONE of the following describes the difference between the measured reactor coolant temperature and the temperature at which the coolant will boil?

A. The subcooled margin is 115 °F B. The subcooled margin is 111 °F C. The superheated margin is 111 °F D. The superheated margin is 115 °F Proposed Answer: B Explanation (Optional):

Candidate must recognize question is asking for subcooling margin by knowing its definition.

Then, candidate determines saturation temp for current RCS pressure (add 15 psi to 2185 psig to obtain 2200 psia and resultant sat temp of 649 °F). Candidate must also know that subcooling margin is calculated using CETs, not hot leg, cold leg, or avg temps. Subcooling margin is then 649 - 538 = 111 °F. The value of 115 °F was used as a distractor as it corresponds to using the value of Thot instead of CETs.

Technical

Reference:

Steam Tables Proposed references to be provided to applicants during examination:

Steam Tables Learning Objective: (As available)

Question Source: Bank #

Modified Bank # STP - 780 (Note changes or attach parent)

New Question History: Last NRC Exam Cognitive Level: Memory or Fundamental Knowledge Comprehension or Analysis X

10 CFR Part 55 Content: 55.41 14 55.43 Comments:

KA: 056 Loss of Offsite Power, AK1.03 - Knowledge of the operational implications of the following concepts as they apply to Loss of Offsite Power.

ES-401 Written Examination Question Worksheet Form ES-401-5 NUREG-1021 Revision-9 Examination Outline Cross-reference: Level RO SRO Tier # 1 Group # 1 K/A # 058.AA2.03 Importance Rating 3.5 Proposed Question 15:

Unit 1 is in Mode 5.

  • RCS Temperature: 170 °F
  • RCS Pressure: 380 psig
  • RHR Trains A and B in service A problem occurs causing the RCS temperature to decrease. The Primary RO observes:
  • Train A RHR Heat Exchanger Bypass Valve (FCV-0851) is full closed
  • Train A RHR Heat Exchanger Flow Control Valve (HCV-0864) is full open
  • RHR Train B operating normally Which ONE of the following failures accounts for these indications?

A. Loss of 125 VDC Bus E1A11 B. Loss of 125 VDC Bus E1D11 C. Loss of 480 MCC E1B1 D. Loss of Instrument Air to Containment Proposed Answer: A Explanation (Optional):

A. Correct - See Addendum 1 to POP04-DJ-0001 B. Incorrect - Loss of this bus does not affect RHR Train A C. Incorrect - Loss of this bus affects RHR Train B, not Train A D. Incorrect - Loss of IA to containment would affect both trains as described in stem, not just the "A" train 0POP04-DJ-0001, Loss Of Class 1E 125 VDC Power (Rev 16);

Technical

Reference:

LOT201.09.HO.01, RHR System (Rev. 11, Page 10-11)

Proposed references to be provided to applicants during examination:

None Learning Objective: (As available)

Question Source: Bank # STP - 358 Modified Bank # (Note changes or attach parent)

New Question History: Last NRC Exam 2003 Cognitive Level: Memory or Fundamental Knowledge Comprehension or Analysis X 10 CFR Part 55 Content: 55.41 7 55.43 Comments:

KA: 058 Loss of DC Power, AA2.03 - Ability to determine and interpret the following as they apply to Loss of DC Power: DC loads lost; impact on ability to operate and monitor plant systems

ES-401 Written Examination Question Worksheet Form ES-401-5 NUREG-1021 Revision-9 Examination Outline Cross-reference: Level RO SRO Tier # 1 Group # 1 K/A # 062.G2.1.28 Importance Rating 3.2 Proposed Question 16:

Unit 1 is operating at 100% power when a Large Break LOCA occurs. Five minutes later, the Primary Operator notices the following:

  • ECW Pump 1B is running
  • ECW Train 1B Blowdown Isolation Valve is closed
  • ECW Train 1B Screen Wash Booster Pump is running
  • ECW Pump 1B Discharge Valve indicates intermediate position (red AND green lights lit)
  • ECW Trains A and C are operating normally
  • The yard watch reports the ECW Pump 1B Discharge Valve is 50% open Which ONE of the following is true concerning ECW Train 1B?

A. Safety Injection Train B was reset prior to the discharge valve reaching full open.

The discharge valve will open fully when the control switch is taken to OPEN.

B. Safety Injection Train B did not actuate. Manually actuating Safety Injection will open the discharge valve fully.

C. ECW Pump 1B did not receive a start signal from the sequencer. The pump was running prior to the Large Break LOCA.

D. Safety Injection actuation has blocked the trip of the pump to allow the train to operate. The pump will continue to run even if the discharge valve is partially closed.

Proposed Answer: D Explanation (Optional):

A. INCORRECT - If SI was reset after 5 minutes with the discharge valve not full open, the pump would trip B. INCORRECT - If SI was not actuated on Train B, then the pump could not be running due to the partially closed discharge valve C. INCORRECT - The pump could not have been running in this condition prior to the LBLOCA D. CORRECT - An SI actuation will block the trip of an ECW pump from discharge valve position LOT201.13, Essential Cooling Water & Ventilation System (Rev. 9);

Technical

Reference:

Electrical drawings 9E-EW01-01, 9E-EW04-02 Proposed references to be provided to applicants during examination:

None Learning Objective: (As available)

Question Source: Bank # STP - 19 Modified Bank # (Note changes or attach parent)

New Question History: Last NRC Exam 2001 Cognitive Level: Memory or Fundamental Knowledge Comprehension or Analysis X 10 CFR Part 55 Content: 55.41 7 55.43 Comments:

Replaced original KA (G2.3.4) - rad exposure limits N/A to ECW system and 2.3.4 is tested in Tier 3, Generic K&A Categories KA: 062 Loss of Nuclear Service Water, G2.1.28 - Knowledge of the purpose and function of major system components and controls STP - has Essential Cooling Water, not Nuclear Service Water

ES-401 Written Examination Question Worksheet Form ES-401-5 NUREG-1021 Revision-9 Examination Outline Cross-reference: Level RO SRO Tier # 1 Group # 1 K/A # 065.AK3.08 Importance Rating 3.7 Proposed Question 17:

Unit 1 is in Mode 4, coming out of a refueling outage, when a loss of instrument air occurs.

Given the following:

  • Narrow Range SG levels; 1A = 40%, 1B = 35%, 1C = 55%, 1D = 20%
  • RHR Train 1B is in service and supplying Low Pressure Letdown
  • RHR Train 1A and 1C are available
  • SI Pumps are in the normal Mode 4 alignment
  • Three Circulating Water pumps are running Which ONE of the following RHR valve alignment methods are Control Room operators directed to use for RCS temperature control in accordance with 0POP04-IA-0001, Loss of Instrument Air, Addendum 6, RCS Temperature Control on RHR?

A. Cycle LOOP B TC INJ MOV-0031B and RHR B Heat Exchanger OUTL TEMP CONT HCV-0865 as needed for temperature control.

B. Cycle MINI FLOW MOV-0067B and RHR Heat Exchanger BYP FLOW CONT FCV-0852 as needed for temperature control.

C. Cycle MINI FLOW MOV-0067B and LOOP B TC INJ MOV-0031B as needed for temperature control.

D. Cycle LOOP B TC INJ MOV-0031B and RHR B Heat Exchanger CCW OUTL FV-4548 as needed for temperature control.

Proposed Answer: C Explanation (Optional):

A: INCORRECT - RHR B Heat Exchanger OUTL TEMP CONT HCV-0865 fails open on a loss of IA, regardless of control room handswitch position. This would result in full RHR cooling.

B: INCORRECT - RHR B Heat Exchanger BYP FLOW CONT FCV-0852 fails closed on a loss of IA, regardless of control room handswitch position. RHR flow may or may not be aligned to the RCS, dependent upon RH-MOV-0031B position.

C: CORRECT - This is the correct alignment per Addendum 6 of 0POP04-IA-0001.

D: INCORRECT - Cycling the CCW supply valve to the RHR Heat Exchanger would cause

undesirable pressure and flow transients on the CCW system and the RHR Heat Exchanger, as well as creating thermal stresses on the heat exchanger tubes.

0POP04-IA-0001, Loss Of Instrument Air (Rev 11), Addendum 6 Technical

Reference:

RCS Temperature Control on RHR Proposed references to be provided to applicants during examination:

None Learning Objective: (As available)

Question Source: Bank # STP - 26 Modified Bank # (Note changes or attach parent)

New Question History: Last NRC Exam Cognitive Level: Memory or Fundamental Knowledge Comprehension or Analysis X 10 CFR Part 55 Content: 55.41 10 55.43 Comments:

Difficulty Justification: The candidate must apply specific RHR system knowledge to determine the failure modes of the RHR air operated valves. Furthermore, the candidate must apply this knowledge to analyze which of the RHR system alignments will properly control RCS temperature, under the given conditions, without challenging the integrity of the RHR or CCW systems.

Replaced original KA (AK3.04) - IA/SA tested by two systems K&As (078.K1.02 and 079.A4.01)

KA: 065 Loss of Instrument Air, AK3.08 - Knowledge of the reasons for the following responses as they apply to the Loss of Instrument Air: Actions contained in EOP for loss of instrument air

ES-401 Written Examination Question Worksheet Form ES-401-5 NUREG-1021 Revision-9 Examination Outline Cross-reference: Level RO SRO Tier # 1 Group # 1 K/A # W/E11.EK3.2 Importance Rating 3.5 Proposed Question 18:

During the performance of 0POP05-EO-EC11, Loss of Emergency Coolant Recirculation, SGs are first depressurized at maximum rate to 735 psig, then more slowly to 255 psig.

Depressurizing slowly to 255 psig is designed to extend the time for:

A. Accumulator Injection while minimizing Nitrogen injection into the RCS.

B. RWST depletion while maintaining cold leg injection.

C. HHSI injection before the larger capacity LHSI pumps are allowed to inject.

D. SG inventory depletion until RHR can be placed in service.

Proposed Answer: A Explanation (Optional):

0POP05-EO-EC11 Step 31 directs operators to depressurize SGs at maximum rate using steam dumps or SG PORV to 735 psig. Step 32 directs operators to slowly depressurize SGs to 255 psig to extend the time of accumulator injection and to minimize the nitrogen injection into the RCS after accumulators are depleted. This makes A the only correct answer.

Technical

Reference:

0POP05-EO-EC11 (Rev 13, pages 26-28)

Proposed references to be provided to applicants during examination:

None Learning Objective: (As available)

Question Source: Bank #

Modified Bank # (Note changes or attach parent)

New X Question History: Last NRC Exam Cognitive Level: Memory or Fundamental Knowledge X Comprehension or Analysis 10 CFR Part 55 Content: 55.41 10 55.43 Comments:

KA: W/E11 Loss of Emergency Coolant Recirculation, EK3.2 - Normal, abnormal and emergency operating procedures associated with Loss of Emergency Coolant Recirculation

ES-401 Written Examination Question Worksheet Form ES-401-5 NUREG-1021 Revision-9 Examination Outline Cross-reference: Level RO SRO Tier # 1 Group # 2 K/A # 028.AK3.02 Importance Rating 2.9 Proposed Question 19:

Given the following conditions:

  • The plant is at 100% power. All control systems are in automatic.
  • Steady state conditions exist.
  • The controlling pressurizer level channel, LT-465, slowly fails high.

Without operator action, which ONE (1) of the following describes the response of charging and letdown?

A. Charging flow will decrease due to the level channel failure, and the letdown isolation valve, LCV-465, will remain open.

B. Charging flow will decrease due to the level channel failure, and the letdown isolation valve, LCV-465, will close.

C. Charging flow will increase due to the level channel failure, and the letdown isolation valve, LCV-465, will remain open.

D. Charging flow will increase due to the level channel failure, and the letdown isolation valve, LCV-465, will close.

Proposed Answer: B Explanation (Optional):

A. Incorrect - response of charging correct, response of letdown incorrect (LCV-465 closes on actual PZR low level due to letdown isolation and reduced charging)

B. Correct C. Incorrect - response of charging incorrect, response of letdown incorrect D. Incorrect - response of charging incorrect, response of letdown correct LOT201.06.HO.01, CVCS (Rev. 12, PAGE 10 of 48);

Technical

Reference:

0POP04-RP-0002, Loss Of Automatic Pressurizer Level Control (Rev 17)

Proposed references to be provided to applicants during examination:

None Learning Objective: (As available)

Question Source: Bank # CPSES Modified Bank # (Note changes or attach parent)

New Question History: Last NRC Exam Cognitive Level: Memory or Fundamental Knowledge Comprehension or Analysis X 10 CFR Part 55 Content: 55.41 7

55.43 Comments:

KA: 028 Pressurizer Level Control Malfunction, AK3.02 - Knowledge of the reasons for the following responses as they apply to the Pressurizer Level Control Malfunction: Relationships between PZR pressure increase and reactor makeup/letdown imbalance.

Modified distractors (Bank answers A, C, and D) by changing the effect on the letdown system from "in-service letdown orifice isolation valve will close" to "letdown isolation valve will remain open" to be more credible.

ES-401 Written Examination Question Worksheet Form ES-401-5 NUREG-1021 Revision-9 Examination Outline Cross-reference: Level RO SRO Tier # 1 Group # 2 K/A # 032.AK2.01 Importance Rating 2.7*

Proposed Question 20:

A reactor startup is in progress with IR power at 3 E-11 amps. The source range High Flux Trip has NOT been blocked.

Which ONE of the following describes the Reactor Protection System response if a CONTROL POWER fuse blows on Source Range N31 with Source Range N31 Trip Bypass Switch in the positions indicated?

NORMAL BYPASS A. No Trip No Trip B. Reactor Trip No Trip C. No Trip Reactor Trip D. Reactor Trip Reactor Trip Proposed Answer: D Explanation (Optional):

Removal of control power fuses: when the control power fuses are removed from the SR, IR, or PR NIS drawer then all the protective trips, and alarms associated with that channel will actuate or alarm and their light boxes will illuminate on CP-005. To meet the action requirements of tech specs (i.e., remove channel from service) the most effective way is to remove the control power fuses. The detector will remain energized and the drawer and control board indication will not change when the control power is removed but all the protective functions will be tripped. If a source range or intermediate range channel is in "level trip bypass" the bypass circuit gets its power from control power, therefore if the control fuses were removed with the channel in "trip bypass" the reactor will trip from a high flux signal.

LOT201.16.HO.01, Excore Nuclear Instrumentation Student Technical

Reference:

Handout (Rev. 12, Page 31 of 42)

Proposed references to be provided to applicants during examination:

None Learning Objective: (As available)

Question Source: Bank # STP - 754 Modified Bank # (Note changes or attach parent)

New Question History: Last NRC Exam

Cognitive Level: Memory or Fundamental Knowledge Comprehension or Analysis X 10 CFR Part 55 Content: 55.41 2 55.43 Comments:

KA: 032 Loss of Source Range Nuclear Instrumentation, AK2.01 - Knowledge of the interrelations between the Loss of Source Range Nuclear Instrumentation and the following:

Power supplies, including proper switch positions.

ES-401 Written Examination Question Worksheet Form ES-401-5 NUREG-1021 Revision-9 Examination Outline Cross-reference: Level RO SRO Tier # 1 Group # 2 K/A # 036.AA2.02 Importance Rating 3.4 Proposed Question 21:

Unit 1 is in Mode 6. You are observing core offload activities in progress. Two spent fuel assemblies have been transferred to the Spent Fuel Pool and a third has just started to be withdrawn from the core.

Which ONE of the following is NOT an indication of a fuel-handling incident?

A. 68 ft RCB Area Monitors alarming B. MAB Ventilation Monitors alarming C. Gas bubbles rising from the refueling cavity D. An announcement to evacuate the Fuel Handling Building Proposed Answer: B Explanation (Optional):

A. Incorrect - This is a symptom or entry condition to 0POP04-FH-0001, Fuel Handing Accident B. Correct - This could only occur if the PAL is open, which it is not during offloading activities C. Incorrect - This is a symptom or entry condition to 0POP04-FH-0001, Fuel Handing Accident D. Incorrect - This is a symptom or entry condition to 0POP04-FH-0001, Fuel Handing Accident Technical

Reference:

0POP04-FH-0001, Fuel Handing Accident (Rev 7)

Proposed references to be provided to applicants during examination:

None Learning Objective: (As available)

Question Source: Bank #

Modified Bank # (Note changes or attach parent)

New X Question History: Last NRC Exam N/A Cognitive Level: Memory or Fundamental Knowledge X Comprehension or Analysis 10 CFR Part 55 Content: 55.41 10, 12 55.43 Comments:

KA: 036 Fuel Handling Accident, AA2.02 - Ability to determine and interpret the following as

they apply to the following: occurrence of a fuel handling incident ES-401 Written Examination Question Worksheet Form ES-401-5 NUREG-1021 Revision-9 Examination Outline Cross-reference: Level RO SRO Tier # 1 Group # 2 K/A # 037.AK1.01 Importance Rating 2.9*

Proposed Question 22:

0POP05-EO-EO30, Steam Generator Tube Rupture, is in progress and an RCS cooldown is desired.

  • Ruptured SG pressure is 900 psig.
  • Containment pressure = 0.3 psig
  • Containment radiation levels are normal Given the above plant conditions, which ONE of the below is the REQUIRED target temperature for RCS cooldown to ensure approximately 50 ºF subcooling during subsequent RCS depressurization AND the specified indication used for determining RCS temperature during the cooldown?

A. 486 °F by RCS WR Hot Leg temperature because of adverse containment conditions.

B. 486 °F by Core Exit Thermocouples temperature because of normal containment conditions.

C. 471 °F by Core Exit Thermocouples temperature because of adverse containment conditions.

D. 471 °F by RCS WR Hot Leg temperature because of normal containment conditions.

Proposed Answer: B Explanation (Optional):

Candidate must convert psig to psia (900 + 15 = 915 psia), then calculate Tsat for 915 psia using saturated steam tables (534 °F). Given desired subcooling of 50 °F, answer is 534-50=484 °F. Core exit T/Cs are used since no adverse containment condition exists.

A. Incorrect - correct temp but incorrect temp indicator (471 °F is used for adverse containment conditions)

B. Correct - both temp indicator and temp are correct C. Incorrect - both temp indicator and temp are incorrect D. Incorrect - temp indicator is correct, temp is incorrect 0POP05-EO-EO30, Steam Generator Tube Rupture (Rev 16);

Technical

Reference:

Steam Tables Proposed references to be provided to applicants during examination: Steam Tables Learning Objective: (As available)

Farley Question Source: Bank #

(from NRC Reg II)

Modified Bank # (Note changes or attach parent)

New Question History: Last NRC Exam N/A Cognitive Level: Memory or Fundamental Knowledge Comprehension or Analysis X 10 CFR Part 55 Content: 55.41 10, 14 55.43 Comments:

KA: 037 Steam Generator Tube Leak, AK1.01 - Knowledge of the operational implications of the following concepts as they apply to SG Tube Leak: Use of steam tables

ES-401 Written Examination Question Worksheet Form ES-401-5 NUREG-1021 Revision-9 Examination Outline Cross-reference: Level RO SRO Tier # 1 Group # 2 K/A # 068.AK2.02 Importance Rating 3.9 Proposed Question 23:

The following plant conditions exist:

  • Mode 1, 48% Reactor power
  • Power ascension in progress
  • A fire occurs requiring an immediate evacuation of the control room
  • The Operators are UNABLE to trip the reactor or perform the other IMMEDIATE ACTIONS of 0POP04-ZO-0001, Control Room Evacuation, before exiting the Control Room Which ONE of the following actions will cause the Solid State Protection System (SSPS) to initiate a reactor trip?

A. Tripping the main turbine from the ASP B. Locally de-energizing 480V Bus 1K1 C. Locally de-energizing 118 VAC Vital Instrumentation Bus I (DP 1201)

D. Tripping any of the RCP breakers at Aux Bus 1F, 1G, 1H, or 1J Proposed Answer: D Explanation (Optional):

A. Incorrect - Since below P-9 (> 50% power), turbine trip will NOT cause Rx trip B. Incorrect - No automatic trip would occur on loss of just one Rod Drive MG set (and 480 V Bus 1L1 would continue to power the other rod drive MG set)

C. Incorrect because no trip signal is generated from a loss of NB02. NE02 would energize the bus.

D. Correct - Since > P-8 (40%), loss of flow in one loop will generate a Rx trip SSPS Student handout (LOT201.20.HO.01)

RCPs Student Handout (LOT201.05.HO.01 Rev. 12)

Technical

Reference:

0POP04-ZO-0001, Control Room Evacuation (Rev 25)

Rod Control System Student HO (LOT201.18.HO.01 Rev. 9)

Proposed references to be provided to applicants during examination:

None Learning Objective: (As available)

Question Source: Bank # Callaway Modified Bank # (Note changes or attach parent)

New Question History: Last NRC Exam Callaway 2002 NRC Exam Cognitive Level: Memory or Fundamental Knowledge

Comprehension or Analysis X 10 CFR Part 55 Content: 55.41 7 55.43 Comments:

KA: 068 Control Room Evacuation, AK2.01 - Knowledge of the interrelations between the Control Room Evacuation and the following: Reactor trip system

ES-401 Written Examination Question Worksheet Form ES-401-5 NUREG-1021 Revision-9 Examination Outline Cross-reference: Level RO SRO Tier # 1 Group # 2 K/A # W/E14.G2.4.16 Importance Rating 3.0 Proposed Question 24:

Unit 1 experienced a Large Break LOCA. The crew just started performing 0POP05-EO-FRZ1, Response To High Containment Pressure, due to an ORANGE path on Containment Pressure.

  • 1M02/D1, RWST LO-LO/EMPTY, has just actuated Which ONE of the following should be performed?

A. CONTINUE in FRZ1 until completed, then transition to 0POP05-EO-ES13, Transfer To Cold Leg Recirculation. Upon completion, transition to 0POP05-EO-EO10, Loss Of Reactor or Secondary Coolant.

B. CONTINUE in FRZ1 until Steps 1-6 are completed, then transition to 0POP05-EO-ES13, Transfer To Cold Leg Recirculation. Complete ES13 Steps 1-6, then transition back to FRZ1.

C. SUSPEND performance of FRZ1, transition to 0POP05-EO-ES13, Transfer To Cold Leg Recirculation. Complete ES13 Steps 1-6, then transition to 0POP05-EO-EO10, Loss Of Reactor or Secondary Coolant.

D. SUSPEND performance of FRZ1, transition to 0POP05-EO-ES13, Transfer To Cold Leg Recirculation. Complete ES13 Steps 1-6, then return to FRZ1.

Proposed Answer: D Explanation (Optional):

EOP Users Guide states certain contingency EOPs take precedence over the FRPs due to specific initiating events. One of these events is RWST level reaching the switchover point to cold leg recirc. (Thus A and B are incorrect). ES13 contains a note before Step 7 that states Function Restoration procedures may now be implemented. Since operators have not completed FRZ1, they would transition back to it, not EO10. Thus C is incorrect, and D is the only correct answer.

0POP01-ZA-0018, Emergency Operating Procedure Users Guide Technical

Reference:

(Rev 17, Section 7.6);

0POP05-EO-ES13, Transfer to Cold Leg Recirculation (Rev 8)

Proposed references to be provided to applicants during examination:

None Learning Objective: (As available)

Question Source: Bank # STP - 151 Modified Bank # (Note changes or attach parent)

New

Question History: Last NRC Exam Cognitive Level: Memory or Fundamental Knowledge Comprehension or Analysis X 10 CFR Part 55 Content: 55.41 10 55.43 Comments:

KA: G2.4.16 - Knowledge of EOP implementation hierarchy and coordination with other support procedures.

Replace Exam Bank Q151 Distractor C since it is a combination of B and some of D (now Distractor B). Also, changed second sentence in Q151 Distractor B from Complete ES13 Steps 1-6, then transition back to E0 to Complete ES13 Steps 1-6, then transition to 0POP05-EO-EO10, Loss Of Reactor or Secondary Coolant (now Distractor C).

ES-401 Written Examination Question Worksheet Form ES-401-5 NUREG-1021 Revision-9 Examination Outline Cross-reference: Level RO SRO Tier # 1 Group # 2 K/A # W/E02.EA2.1 Importance Rating 3.3 Proposed Question 25:

Given the following Unit 2 conditions:

  • A Small Break LOCA has occurred
  • SI has been reset
  • Operators have just completed Step 1 of 0POP05-EO-EO10, Loss of Reactor or Secondary Coolant The Shift Technical Advisor reports the following:
  • RCS pressure - 1830 psig and stable
  • RCS temperature - 550 ºF and stable
  • RCS subcooling - 60 °F
  • Pressurizer level - 15%
  • SG A, B, C and D NR Levels are 8%, 10%, 17% and 19% respectively
  • Total AFW flow - 400 gpm
  • Adverse containment conditions do NOT exist Which ONE of the following actions will the operators perform?

A. Manually actuate SI and transition to 0POP05-EO-EO00, Reactor Trip or Safety Injection B. Transition to 0POP05-EO-ES11, SI Termination C. Transition to 0POP05-EO-FRP2, Response to Anticipated Pressurized Thermal Shock Condition D. Transition to 0POP05-EO-FRH1, Response to Loss of Secondary Heat Sink Proposed Answer: B Explanation (Optional):

A: INCORRECT - Given conditions do not meet the requirements for SI reinitiation, and if it did, SI is reinitiated by manually starting SI pumps, not manually actuating SI.

B: CORRECT - The given conditions would allow transition to ES11, which would be the expected action.

C: INCORRECT - Given conditions do not meet the entry requirements for FRP2 D: INCORRECT - Total SG flow can be less than 576 gpm if one SG level is greater than 14% NR 0POP05-EO-EO10, Loss of Reactor or Secondary Coolant (Rev 17, Technical

Reference:

Conditional Information Page)

Proposed references to be provided to applicants during examination:

None Learning Objective: 81103 (As available)

Question Source: Bank # STP - 52 Modified Bank # (Note changes or attach parent)

New Question History: Last NRC Exam Cognitive Level: Memory or Fundamental Knowledge Comprehension or Analysis X 10 CFR Part 55 Content: 55.41 10 55.43 Comments:

KA: W/E02 SI Termination, EA2.1 - Ability to determine and interpret the following as they apply to SI Termination: Facility conditions and selection of appropriate procedures during abnormal and emergency operations.

ES-401 Written Examination Question Worksheet Form ES-401-5 NUREG-1021 Revision-9 Examination Outline Cross-reference: Level RO SRO Tier # 1 Group # 2 K/A # W/E13.EA1.2 Importance Rating 3.0 Proposed Question 26:

Operators are performing 0POP05-EO-FRH2 in response to a Steam Generator Overpressure event. Due to plant conditions, the Unit Supervisor has directed steam to be dumped from the affected SG via local operation of its PORV.

A CAUTION preceding Step 1 of the SG PORV Local Operation Addendum states SG PORVs should NOT be opened GREATER THAN 50%.

Which ONE of the following identifies the reason for this CAUTION?

A. To prevent exceeding the maximum cooldown rate of < 100 °F / HR.

B. Too large of a release of steam will cause SG levels to rise rapidly and cause damage to piping from water hammer.

C. The PORV hydraulic unit accumulators only contain sufficient stored energy for one and one-half strokes.

D. To prevent a Main Steam Isolation signal due to high steam pressure rate drop.

Proposed Answer: C Explanation (Optional):

A. Incorrect - While the procedure does direct cooldown to < 100 °F/HR if necessary, it is not the basis for restricted motion of the PORV.

B. Incorrect - Water hammer can occur if SG level rises too high, but it is not the basis forrestricted motion of the PORV.

C. Correct.

D. A High Steam Pressure Rate ESFAS initiation signal can occur if step decrease of 100 psi or ramp decrease of > 2 psi/sec on 2/3 channels on 1/4 steamline and < P-11 and low compensated steamline pressure blocked, but it is not the basis for restricted motion of the PORV.

0POP05-EO-FRH2, Response to Steam Generator Overpressure Technical

Reference:

(Rev 5, Addendum 1, SG PORV Local Operation)

Proposed references to be provided to applicants during examination:

None Learning Objective: T50434 (As available)

Question Source: Bank #

Modified Bank # (Note changes or attach parent)

New X Question History: Last NRC Exam

Cognitive Level: Memory or Fundamental Knowledge X Comprehension or Analysis 10 CFR Part 55 Content: 55.41 4, 10 55.43 Comments:

KA: W/E13 Steam Generator Overpressure, EA1.2 - Ability to operate and/or monitor the following as they apply to the Steam Generator Overpressure: Operating behavior characteristics of the facility.

ES-401 Written Examination Question Worksheet Form ES-401-5 NUREG-1021 Revision-9 Examination Outline Cross-reference: Level RO SRO Tier # 1 Group # 2 K/A # W/E08.EA2.2 Importance Rating 3.5 Proposed Question 27:

Procedure 0POP05-EO-FRP1, Response to Imminent Pressurized Thermal Shock Condition, contains less restrictive SI termination criteria than other procedures.

Why is it more desirable to terminate SI when in this procedure?

A. SI flow may have contributed to the RCS cooldown.

B. The other SI termination criteria will have already been met when 0POP05-EO-FRP1 is entered.

C. RCS heat removal is via the steam generators and SI flow is NOT required.

D. To conserve water in the RWST.

Proposed Answer: A Explanation (Optional):

A. Correct.

B. Incorrect. Entry requirements into FRP1 are not contingent upon SI termination criteria C. Incorrect. RCS heat removal may or may not be provided by the SGs (feed and bleed).

SI flow may be required for heat removal, and in those cases, the appropriate CSF status tree will dictate priority (and thus SI termination criteria).

D. Incorrect. RWST inventory does not factor into SI termination criteria in FRP1.

0POP05-EO-FRP1, Response to Imminent Pressurized Thermal Technical

Reference:

Shock Condition (Rev 10)

Proposed references to be provided to applicants during examination:

Learning Objective: (As available)

Question Source: Bank # STP - 1191 Modified Bank # (Note changes or attach parent)

New Question History: Last NRC Exam Cognitive Level: Memory or Fundamental Knowledge X Comprehension or Analysis 10 CFR Part 55 Content: 55.41 10 55.43 Comments:

KA: W/E08 Pressurized Thermal Shock, EA2.2 - Ability to determine and interpret the following as they apply to the Pressurized Thermal Shock: Adherence to appropriate

procedures and operation within the limitations in the facilitys license and amendments.

ES-401 Written Examination Question Worksheet Form ES-401-5 NUREG-1021 Revision-9 Examination Outline Cross-reference: Level RO SRO Tier # 2 Group # 1 K/A # 003.K4.04 Importance Rating 2.8 Proposed Question 28:

Which one of the following describes the purpose of the RCP thermal barrier?

A. Provide heatup of pump internals.

B. Prevents cooler seal injection water from creating thermal stresses in the pump impeller assembly.

C. Limits the amount of seal injection water reaching the RCS to limit inventory loss.

D. Limit heat flow from the RCS water to the radial bearing and Thermal Barrier Heat Exchanger.

Proposed Answer: D Explanation (Optional):

Technical

Reference:

LOT201.05.HO.01 REV 12 (page 4 of 30)

Proposed references to be provided to applicants during examination:

None Learning Objective: (As available)

Question Source: Bank #

Modified Bank # (Note changes or attach parent)

New X Question History: Last NRC Exam Cognitive Level: Memory or Fundamental Knowledge X Comprehension or Analysis 10 CFR Part 55 Content: 55.41 3 55.43 Comments:

KA: 003 Reactor Coolant Pump System, K4.04 - Knowledge of RCPS design feature(s) and/or interlocks which provide for the following: Adequate cooling of RCP motor and seals

ES-401 Written Examination Question Worksheet Form ES-401-5 NUREG-1021 Revision-9 Examination Outline Cross-reference: Level RO SRO Tier # 2 Group # 1 K/A # 003.A2.03 Importance Rating 2.7 Proposed Question 29:

While operating at full power a high RCP Motor Stator Winding Temperature Alarm is received. Assuming this is a valid alarm, what operator action is required per 0POP04-RC-0002, Reactor Coolant Pump Off Normal, and what is the concern for a high stator winding temperature?

A. Trip the reactor, then stop the affected RCP. Concern is breakdown of winding insulation resistance resulting in shorts/grounds.

B. Stop the RCP, then trip the reactor. Concern is breakdown of winding insulation resistance resulting in shorts/grounds.

C. Trip the reactor, then stop the affected RCP. Concern is reduced clearances leading to motor bearing damage.

D. Stop the RCP, then trip the reactor. Concern is reduced clearances leading to motor bearing damage.

Proposed Answer: A Explanation (Optional):

Procedure states if valid alarm for motor stator winding temp, trip Reactor then RCP. Student Handout states if the motor stator is overheated an electrical fault will occur.

0POP04-RC-0002, RCP Off Normal (Rev 22)

Technical

Reference:

LOT201.05.HO.01 Rev. 12, RCP Student Handout Proposed references to be provided to applicants during examination:

None Learning Objective: (As available)

Question Source: Bank #

Modified Bank # (Note changes or attach parent)

New X Question History: Last NRC Exam Cognitive Level: Memory or Fundamental Knowledge X Comprehension or Analysis 10 CFR Part 55 Content: 55.41 3, 10 55.43 Comments:

KA: 003 Reactor Coolant Pump System, A2.03 - Ability to (a) predict the impacts of the following malfunctions or operations on the RCPS; and (b) based on those predictions, use procedures to correct, control, or mitigate the consequences of those malfunctions or operations: Problems associated with RCP motors, including faulty motors and current, and

winding and bearing temperature problems ES-401 Written Examination Question Worksheet Form ES-401-5 NUREG-1021 Revision-9 Examination Outline Cross-reference: Level RO SRO Tier # 2 Group # 1 K/A # 004.A1.06 Importance Rating 3.0 Proposed Question 30:

Given the following:

  • Unit 1 is operating steady state at 93% power
  • LI-112, VCT Level, indicates 35% and decreasing (CP-004)
  • LI-113, VCT Level, indicates 100% (ERFDADS)

Which ONE the following describes the expected plant response?

A. Pressurizer level will decrease to 17% resulting in letdown isolation.

B. Auto makeup will initiate to the VCT when VCT level decreases to 28% and raise VCT level back to 48%.

C. VCT level will continue to decrease until the operator manually aligns Divert valve LCV-112A, to the VCT position.

D. Operating CCP suction will automatically align to the RWST when VCT level decreases to 3%.

Proposed Answer: B Explanation (Optional):

LCV-112A is controlled by VCT level instrument LT-112. As a backup, VCT level instrument LT-113 will override the control signal from channel LT-112 to place LCV-112A in the full divert position if the VCT level increases to 95%. As long as LT-112 is working, normal VCT operations will result. Under steady states ops as stated in the question stem, VCT auto makeup will begin at 28% and decreasing.

Technical

Reference:

LOT201.06.HO.01 CVCS Student Handout (Rev 12)

Proposed references to be provided to applicants during examination:

None Learning Objective: (As available)

Question Source: Bank # STP - 675 Modified Bank # (Note changes or attach parent)

New Question History: Last NRC Exam Cognitive Level: Memory or Fundamental Knowledge Comprehension or Analysis X 10 CFR Part 55 Content: 55.41 5

55.43 Comments:

KA: 004 CVCS, A1.06 - Ability to predict and/or monitor changes in parameters (to prevent exceeding design limits) associated with operating the CVCS controls including: VCT level

ES-401 Written Examination Question Worksheet Form ES-401-5 NUREG-1021 Revision-9 Examination Outline Cross-reference: Level RO SRO Tier # 2 Group # 1 K/A # 005.K2.01 Importance Rating 3.0 Proposed Question 31:

Unit 2 is in Mode 4 with the following conditions:

  • RCS temp is 335F and lowering
  • RCS pressure is 330 psig
  • A train 4.16 KV / 480V ESF transformer has failed Which of the following RHR pumps would be available for continued plant cooldown?

A. 1A and 1B only B. 1B and 1C only C. 1A and 1C only D. 1A and 1B and 1C Proposed Answer: B Explanation (Optional):

RHR Pump 1A is powered from LC E1A2. Therefore, loss of this load center due to a transformer failure would make pumps 1B and 1C available for plant cooldown.

Technical

Reference:

LOT201.09.HO.01 Rev. 11 (page 6 of 24)

Proposed references to be provided to applicants during examination:

None Learning Objective: (As available)

Question Source: Bank #

Modified Bank # (Note changes or attach parent)

New X Question History: Last NRC Exam Cognitive Level: Memory or Fundamental Knowledge X Comprehension or Analysis 10 CFR Part 55 Content: 55.41 7 55.43 Comments:

KA: 005 RHR System, K2.01 - Knowledge of bus power supplies to the following: RHR pumps

ES-401 Written Examination Question Worksheet Form ES-401-5 NUREG-1021 Revision-9 Examination Outline Cross-reference: Level RO SRO Tier # 2 Group # 1 K/A # 006.K5.08 Importance Rating 2.9*

Proposed Question 32:

An SI has initiated due to a LBLOCA. When RCS pressure reaches 200 psig and steady, LHSI Pump 1A trips.

What change in total ECCS flow rate and RCS pressure would the operator expect to observe?

A. Total ECCS flow rate increases and RCS pressure decreases B. Total ECCS flow rate decreases and RCS pressure increases C. Total ECCS flow rate decreases and RCS pressure does not change D. Total ECCS flow rate increases and RCS pressure does not change Proposed Answer: C Explanation (Optional):

At 200 psig, all accumulators have completed injection and HHSI pumps and LHSI pumps are running. Tripping one LHSI pump will reduce ECCS flow rate but will not affect RCS pressure. Student must be knowledgeable of theory of centrifugal pumps operating in parallel and apply it to the given scenario.

Technical

Reference:

LOT201.10 ECCS.ppt (slide 31)

Proposed references to be provided to applicants during examination:

None Learning Objective: (As available)

Question Source: Bank #

Modified Bank # (Note changes or attach parent)

New X Question History: Last NRC Exam Cognitive Level: Memory or Fundamental Knowledge Comprehension or Analysis X 10 CFR Part 55 Content: 55.41 8 55.43 Comments:

KA: 006 ECCS, K5.08 - Knowledge of the operational implications of the following concepts as they apply to ECCS: Operation of pumps in parallel

ES-401 Written Examination Question Worksheet Form ES-401-5 NUREG-1021 Revision-9 Examination Outline Cross-reference: Level RO SRO Tier # 2 Group # 1 K/A # 006.A2.04 Importance Rating 3.4 Proposed Question 33:

Unit 2 is in Mode 3 with Pressurizer pressure = 2050 psig. The ACC TK 2B PRESS HI/LO alarm has actuated on 1M02. Accumulator 2B pressure = 610 psig.

What would be the effect on plant safety if Accumulator 2B pressure were allowed to decrease below its Tech Spec limit and how is pressure restored to the allowable operating range?

A. The Accumulator injection rate cannot be assumed to provide adequate core cooling during a LOCA; pressurize accumulator using HHSI pump.

B. A sufficient volume of water cannot be assumed to reach the core during a LOCA; pressurize accumulator using HP Nitrogen.

C. The Accumulator injection rate cannot be assumed to provide adequate core cooling during a LOCA; pressurize accumulator using HP Nitrogen.

D. A sufficient volume of water cannot be assumed to reach the core during a LOCA; pressurize accumulator using HHSI pump.

Proposed Answer: B Explanation (Optional):

TS 3.5.1, Accumulators Technical

Reference:

LOT201.10, ECCS Student Handout 0POP02-SI-0001, SI Accumulators (Rev 22)

Proposed references to be provided to applicants during examination:

None Learning Objective: (As available)

Question Source: Bank #

Modified Bank # (Note changes or attach parent)

New X Question History: Last NRC Exam Cognitive Level: Memory or Fundamental Knowledge X Comprehension or Analysis 10 CFR Part 55 Content: 55.41 7 55.43 Comments:

KA: 006 ECCS, A2.04 - Ability to (a) predict the impacts of the following malfunctions or operations on the ECCS; and (b) based on those predictions, use procedures to correct, control, or mitigate the consequences of those malfunctions or operations: Improper discharge pressure

ES-401 Written Examination Question Worksheet Form ES-401-5 NUREG-1021 Revision-9 Examination Outline Cross-reference: Level RO SRO Tier # 2 Group # 1 K/A # 007.K4.01 Importance Rating 2.6 Proposed Question 34:

Unit 1 is in Mode 1. If a Pressurizer PORV inadvertently opened, at what temperature would the operator expect the fluid entering the PRT to be at assuming the PRT pressure is at 5 psig and what would be the fastest method of cooling the PRT?

A. 228 °F, feed Reactor Makeup Water and bleed to LWPS B. 228 °F, recirculation through RCDT Heat Exchanger C. 162 °F, feed Reactor Makeup Water and bleed to LWPS D. 162 °F, recirculation through RCDT Heat Exchanger Proposed Answer: A Explanation (Optional):

1. 228 °F is the saturation temp for 5 psig
2. Feed and bleed reduces water temp from 200 °F to 120 °F in approx 1 hour1.157407e-5 days <br />2.777778e-4 hours <br />1.653439e-6 weeks <br />3.805e-7 months <br /> while using the RCDT heat exchanger takes approx 8 hrs Technical

Reference:

LOT20104.HO.01 (Rev 7), PZR, PRT, and RCDT Student Handout Proposed references to be provided to applicants during examination:

Steam Tables Learning Objective: (As available)

Question Source: Bank #

Modified Bank # (Note changes or attach parent)

New X Question History: Last NRC Exam Cognitive Level: Memory or Fundamental Knowledge Comprehension or Analysis X 10 CFR Part 55 Content: 55.41 3 55.43 Comments:

KA: 007 PZR Relief Tank / Quench Tank System, K4.01 - Knowledge of PRTS design feature(s) and/or interlock(s) which provide for the following: Quench tank cooling

ES-401 Written Examination Question Worksheet Form ES-401-5 NUREG-1021 Revision-9 Examination Outline Cross-reference: Level RO SRO Tier # 2 Group # 1 K/A # 008.A4.07 Importance Rating 2.9*

Proposed Question 35:

Unit 1 is operating at 100% power when a leak develops in the Component Cooling Water System. Level in the CCW Surge Tank is 65% and decreasing slowly.

0POP04-CC-0001, Loss of Component Cooling Water, is entered.

The operator determines CCW Surge Tank Makeup Valve LV-4501 is NOT open. In accordance with 0POP04-CC-0001, Loss of Component Cooling Water, what action is required?

A. No action is required. CCW Surge Tank level is above the level the valve is expected to automatically open.

B. If the valve cannot be opened, take actions to initiate makeup from the Chemical Addition Tank.

C. If the valve cannot be opened, take actions to initiate makeup from the Reactor Makeup Water System.

D. Trip the reactor, then trip the RCPs due to loss of CCW cooling to the RCPs.

Proposed Answer: C Explanation (Optional):

POP04-CC-0001 states if surge tank level cannot be maintained between 69-74% using LV-4501, an operator is to be dispatched to maintain level using CCW SURGE TANK 1A REACTOR MAKEUP WATER SUPPLY VALVE and to ensure a Reactor Makeup Water pump is operating.

Technical

Reference:

POP04-CC-0001, Loss of CCW (Rev 13, page 3)

Proposed references to be provided to applicants during examination:

None Learning Objective: (As available)

Question Source: Bank #

Modified Bank # (Note changes or attach parent)

New X Question History: Last NRC Exam Cognitive Level: Memory or Fundamental Knowledge Comprehension or Analysis X 10 CFR Part 55 Content: 55.41 10 55.43 Comments:

KA: 008 CCWS, A4.07 - Ability to manually operate and/or monitor in the control room:

Control of minimum level in the CCWS surge tank ES-401 Written Examination Question Worksheet Form ES-401-5 NUREG-1021 Revision-9 Examination Outline Cross-reference: Level RO SRO Tier # 2 Group # 1 K/A # 008.G2.1.27 Importance Rating 2.8 Proposed Question 36:

Which ONE of the following ESF loads is directly cooled by CCW?

A. ESF DG Jacket Cooling Water B. Reactor Containment Fan Coolers (RCFCs)

C. Spent Fuel Pool Heat Exchangers D. HHSI Pump lube oil coolers Proposed Answer: B Explanation (Optional):

A: Incorrect. This load is an ESF component thats cooled by ECW, not CCW B: Correct C: Incorrect. This is a non-ESF load cooled by CCW D: Incorrect. This is an ESF pump, but has no lube oil cooler.

Technical

Reference:

LOT201.12.HO.01, CCW System Student Handout (Rev 11)

Proposed references to be provided to applicants during examination:

None Learning Objective: (As available)

Question Source: Bank #

Modified Bank # (Note changes or attach parent)

New X Question History: Last NRC Exam Cognitive Level: Memory or Fundamental Knowledge X Comprehension or Analysis 10 CFR Part 55 Content: 55.41 7 55.43 Comments:

KA: 008 CCWS, G2.1.27 - Knowledge of system purpose and / or function

ES-401 Written Examination Question Worksheet Form ES-401-5 NUREG-1021 Revision-9 Examination Outline Cross-reference: Level RO SRO Tier # 2 Group # 1 K/A # 010.A2.02 Importance Rating 3.9 Proposed Question 37:

The following plant conditions exist:

  • Unit 1 is performing a plant startup and power ascension
  • Reactor power is at 22%
  • Pressurizer Spray Valve PCV-0655B has failed open How is pressurizer pressure trending and what action is required in accordance with 0POP04-RP-0001, Loss of Automatic Pressurizer Pressure Control?

A. Increasing; trip the reactor and stop RCP 1A only B. Decreasing; trip the reactor and stop RCP 1D only C. Increasing; trip the reactor and stop RCPs 1A & 1D D. Decreasing; trip the reactor and stop RCPs 1A & 1D Proposed Answer: D Explanation (Optional):

Procedure directs reactor trip, turbine trip, stopping RCP 1A and 1D (Step 4)

POP04-RP-0001, Loss of Auto PZR Pressure Control (Rev 12, Technical

Reference:

page 5)

Proposed references to be provided to applicants during examination:

None Learning Objective: (As available)

Question Source: Bank #

Modified Bank # STP - 742 (Note changes or attach parent)

New Question History: Last NRC Exam Cognitive Level: Memory or Fundamental Knowledge Comprehension or Analysis X 10 CFR Part 55 Content: 55.41 10 55.43 Comments:

KA: 010 PZR Pressure Control System, A2.02 - Ability to (a) predict the impacts of the following malfunctions or operations on the PZR PCS; and (b) based on those predictions, use procedures to correct, control, or mitigate the consequences of those malfunctions or

operations: Spray valve failures ES-401 Written Examination Question Worksheet Form ES-401-5 NUREG-1021 Revision-9 Examination Outline Cross-reference: Level RO SRO Tier # 2 Group # 1 K/A # 012.K5.01 Importance Rating 3.3*

Proposed Question 38:

Which ONE of the following Reactor Trip System instrumentation setpoints is designed to protect the reactor core against Departure from Nucleate Boiling?

A. Power Range Positive Rate B. Overpower - Delta T C. Pressurizer High Pressure D. RCP Underfrequency Proposed Answer: D Explanation (Optional):

A: INCORRECT - Rod Ejection B: INCORRECT - Fuel Integrity C: INCORRECT - Overpressure D: CORRECT - DNB Technical

Reference:

TS 2.0, LSSS Bases Proposed references to be provided to applicants during examination:

None Learning Objective: (As available)

Question Source: Bank # STP - 812 Modified Bank # (Note changes or attach parent)

New Question History: Last NRC Exam Cognitive Level: Memory or Fundamental Knowledge X Comprehension or Analysis 10 CFR Part 55 Content: 55.41 14 55.43 Comments:

KA: 012 Reactor Protection System, K5.01 - Knowledge of the operational implications of the following concepts as they apply to the RPS: DNB

ES-401 Written Examination Question Worksheet Form ES-401-5 NUREG-1021 Revision-9 Examination Outline Cross-reference: Level RO SRO Tier # 2 Group # 1 K/A # 012.K6.10 Importance Rating 3.3 Proposed Question 39:

Which ONE of the following describes the effect that a loss of compensating voltage to NI-35 will have on the Solid State Protection System (SSPS)?

A. On a plant shutdown, when P-10 resets an IR High Level Trip will occur.

B. At high power, it will cause an IR/PR Rod Withdrawal Block.

C. On a reactor shutdown, P-6 will clear early and a SR High Flux Trip will occur.

D. On a reactor shutdown, P-6 will NOT clear and reenergize the SR detectors.

Proposed Answer: D Explanation (Optional):

A: Incorrect. Above 1E-8 amps loss of compensating voltage should not cause a visible change in output of the detector/indication, therefore when P-10 resets there should be no consequence.

B: Incorrect. IR/PR Rod Withdrawal Block is not a function processed through the RPS; this function goes between NI to Rod Control. Also above 1E-8 amps loss of compensating voltage should not cause a visible change in output of the detector/indication C: Incorrect. IR NI-35 will remain above 1E-10 amps therefore P-6 will not reset (2/2 channels < 1E-10 amps)

D: Correct. P-6 requires 2/2 IR < 1E-10 amps to reenergize the SR Hi Voltage power supplies and remove the SR High Flux Trip Bypasses. With IR NI-35 undercompensated, it will remain above 1E-10 amps, therefore the SR high Flux Trip will not activate LOT201.16, Excore Nuclear Instrumentation Student HO (Rev 12);

Technical

Reference:

0POP09-AN-05M3 (Rev 23), Window F1; 0POP04-NI-0001, (Rev 4 Addendum 2 step 6)

Proposed references to be provided to applicants during examination:

None Learning Objective: (As available)

Question Source: Bank # STP - 14 Modified Bank # (Note changes or attach parent)

New Question History: Last NRC Exam Cognitive Level: Memory or Fundamental Knowledge Comprehension or Analysis X 10 CFR Part 55 Content: 55.41 2 55.43

Comments:

KA: 012 Reactor Protection System, K6.10 - Knowledge of the effect of a loss or malfunction of the following will have on the RPS: Permissive circuits The student must know the Intermediate Range NIs use Compensated Ion Chambers and understand the effect gamma has on their operation and the operating range affected. The student must also understand the operation/logic of the permissive interlocks and bypasses to conclude that loss of compensating voltage only impacts operation at low levels of the IR.

ES-401 Written Examination Question Worksheet Form ES-401-5 NUREG-1021 Revision-9 Examination Outline Cross-reference: Level RO SRO Tier # 2 Group # 1 K/A # 013.K1.16 Importance Rating 2.9*

Proposed Question 40:

Which ONE of the following groups of setpoints and coincidences will cause a Steam Line Isolation Signal?

A. Steam line pressure less than 735 psig, 2/3 channels on 1/4 steam lines with Safety Injection NOT blocked.

B. Containment HIGH-II pressure greater than 3 psig on 1/3 channels.

C. Steam line pressure less than 735 psig, 2/3 channels on 2/4 steam lines with Safety Injection NOT blocked.

D. Containment HIGH-I pressure greater than 5 psig on 2/3 channels.

Proposed Answer: A Explanation (Optional):

The following provide a main steam isolation signal:

1. Containment High Pressure (HI-2) - 3.0 psig - 2/3 - no permissive
2. High Steam Pressure Rate - step decrease of 100 psi or ramp decrease of >2 psi/sec - 2/3 channels on 1/4 steamlines - < P-11 & low compensated steamline pressure blocked
3. Manual - operator - 1/2 - no permissive
4. Low Steamline Pressure - 735 psig (lead - lag comp.) - 2/3 channels on 1/4 steamlines - >

P-11 or reset/not blocked Technical

Reference:

LOT201.20.HO.02, SSPS Study Guide (Rev 15, page 7)

Proposed references to be provided to applicants during examination:

None Learning Objective: (As available)

Question Source: Bank # STP - 150 Modified Bank # (Note changes or attach parent)

New Question History: Last NRC Exam Cognitive Level: Memory or Fundamental Knowledge X Comprehension or Analysis 10 CFR Part 55 Content: 55.41 7 55.43 Comments:

KA: 013 ESF Actuation System, K1.16 - Knowledge of the physical connections and/or cause-effect relationships between ESFAS and the following systems: MRSS (main and reheat steam system)

ES-401 Written Examination Question Worksheet Form ES-401-5 NUREG-1021 Revision-9 Examination Outline Cross-reference: Level RO SRO Tier # 2 Group # 1 K/A # 022.K4.04 Importance Rating 2.8 Proposed Question 41:

Which of the following conditions are the CRDM cooling fans analyzed to mitigate during a Natural Circulation cooldown in accordance with 0POP05-EO-ES02?

A. Damage to the CRDM coils resulting from overheating.

B. Damage to the ex-core NIS resulting from overheating.

C. Brittle fracture to the reactor vessel head flange welds resulting from exceeding nil ductility temperature limits.

D. Void formation in the reactor upper head area that degrades RCS cooldown capability.

Proposed Answer: D Explanation (Optional):

LOT504.25.HO.01 (Rev 7, page 7);

Technical

Reference:

0POP05-EO-ES02, Natural Circ Cooldown (Rev 9, Step 6)

Proposed references to be provided to applicants during examination:

None Learning Objective: (As available)

Question Source: Bank # STP - 1026 Modified Bank # (Note changes or attach parent)

New Question History: Last NRC Exam 2001 Cognitive Level: Memory or Fundamental Knowledge X Comprehension or Analysis 10 CFR Part 55 Content: 55.41 10 55.43 Comments:

KA: 022 Containment Cooling System, K4.04 - Knowledge of CCS design feature(s) and/or interlock(s) which provide for the following: Cooling of control rod drive motors

ES-401 Written Examination Question Worksheet Form ES-401-5 NUREG-1021 Revision-9 Examination Outline Cross-reference: Level RO SRO Tier # 2 Group # 1 K/A # 026.G2.4.11 Importance Rating 3.4 Proposed Question 42:

0POP05-EO-FRZ1, Response to High Containment Pressure, Step 3.0 checks for conditions that would require containment spray to be initiated. If the conditions are met, the step directs the operator to Verify Containment Spray pumps - RUNNING A CAUTION preceding Step 3.0 states IF 0POP05-EO-EC11, Loss of Emergency Coolant Recirculation, is in effect, THEN Containment Spray should be operated as directed in 0POP05-EO-EC11, Loss of Emergency Coolant Recirculation, rather than Step 3 below.

Why does EC11 take precedence over FRZ1 regarding Containment Spray Pump operation?

A. EC11 ensures containment spray pump suction is aligned to the containment sump.

B. EC11 reduces spray pump operation in order to conserve RWST inventory.

C. EC11 ensures containment fan coolers are running, thus making spray pump operation unnecessary.

D. EC11 reduces spray pump operation since spray has little or no effect on containment heat removal capability during loss of emergency coolant recirculation.

Proposed Answer: B Explanation (Optional):

0POP05-EO-FRZ1, Response to High Containment Pressure (R 6);

Technical

Reference:

0POP05-EO-EC11, Loss of Emergency Coolant Recirc (Rev 13)

Proposed references to be provided to applicants during examination:

None Learning Objective: (As available)

Question Source: Bank # CPSES Modified Bank # (Note changes or attach parent)

New Question History: Last NRC Exam Cognitive Level: Memory or Fundamental Knowledge X Comprehension or Analysis

10 CFR Part 55 Content: 55.41 10 55.43 Comments:

KA: 026 Containment Spray System, G2.4.11 - Knowledge of abnormal condition procedures Objective: FRZ.XH5.OB402, distractors modified. Also, question is from CPSES bank, but reworded by STP

ES-401 Written Examination Question Worksheet Form ES-401-5 NUREG-1021 Revision-9 Examination Outline Cross-reference: Level RO SRO Tier # 2 Group # 1 K/A # 039.K3.05 Importance Rating 3.6 Proposed Question 43:

Unit 2 is at End Of Life (EOL) and is in Mode 3 at normal operating temperature and pressure. A major steam line break occurs upstream of an MSIV. According to the accident analysis for a main steamline break, which ONE of the below depicts the correct sequence of events for this accident analysis?

A. RCS average temperature decreases, pressurizer pressure decreases, core attains criticality, SI injection water reaches the core.

B. Pressurizer pressure decreases, RCS average temperature decreases, core attains criticality, SI injection water reaches the core.

C. Pressurizer pressure decreases, RCS average temperature decreases, SI injection water reaches core, core attains criticality.

D. RCS average temperature decreases, pressurizer pressure decreases, SI injection water reaches core, core attains criticality.

Proposed Answer: A Explanation (Optional):

The decreasing Tave causes Pzr pressure to decrease (0-8 sec). The loop colder water starts entering the core and core average temperature decreases (8-24.8 sec). The core attains criticality and core average temperature decreases at a slower rate (24.8 sec). 2800 ppm boron starts to reach the core slowing the amount of positive reactivity inserted (37.8 sec).

Technical

Reference:

LOT501.16.HO.01 Rev. 1 (Page 10 of 38)

Proposed references to be provided to applicants during examination:

None Learning Objective: (As available)

Question Source: Bank #

Modified Bank # (Note changes or attach parent)

New X Question History: Last NRC Exam Cognitive Level: Memory or Fundamental Knowledge X Comprehension or Analysis 10 CFR Part 55 Content: 55.41 5 55.43 Comments:

KA: 039 Main and Reheat Steam System, K3.05 - Knowledge of the effect that a loss or malfunction of the MRSS will have on the following: RCS

ES-401 Written Examination Question Worksheet Form ES-401-5 NUREG-1021 Revision-9 Examination Outline Cross-reference: Level RO SRO Tier # 2 Group # 1 K/A # 059.K1.02 Importance Rating 3.4*

Proposed Question 44:

Unit 2 is in Mode 2 with AFW NOT in service. Which one of the following correctly identifies the feed flow path downstream of the Low Power FRV?

A. Feedwater Isolation Bypass Valve, auxiliary feed ring B. Feedwater Isolation Bypass Valve, main feed ring C. Preheater Bypass Valve, auxiliary feed ring D. Preheater Bypass Valve, main feed ring Proposed Answer: C Explanation (Optional):

The Preheater Bypass Valves (PBVs) are used to provide a feedwater flow path to the S/Gs during fill and low power operation. The PBV line runs from downstream of the FRVs to the AFW line and ties into it just downstream of the AFW OCIV. Thus C is the only correct answer.

0POP03-ZG-0003, Secondary Plant Startup (Rev 21)

Technical

Reference:

LOT202.13.HO.01, Feedwater System Student Handout (Rev. 10, page 31 of 49)

Proposed references to be provided to applicants during examination:

None Learning Objective: (As available)

Question Source: Bank #

Modified Bank # (Note changes or attach parent)

New X Question History: Last NRC Exam Cognitive Level: Memory or Fundamental Knowledge X Comprehension or Analysis 10 CFR Part 55 Content: 55.41 4 55.43 Comments:

KA: 059 Main Feedwater System, K1.02 - Knowledge of the physical connections and/or cause-effect relationships between the MFW and the following systems: AFW system

ES-401 Written Examination Question Worksheet Form ES-401-5 NUREG-1021 Revision-9 Examination Outline Cross-reference: Level RO SRO Tier # 2 Group # 1 K/A # 061.A3.01 Importance Rating 4.2 Proposed Question 45:

Which ONE of the following statements identifies the expected condition of the AFW system following Loss Of Offsite Power (LOOP) and subsequent completion of the Mode II sequencer?

A. AFW motor-operated pumps and steam driven pump running, total AFW flow to SGs > 576 gpm.

B. AFW motor-operated pumps and steam-driven pump running, total AFW flow to SGs = 0 gpm.

C. Only AFW motor-operated pumps running, total AFW flow to SGs > 576 gpm.

D. Only AFW motor-operated pumps running, total AFW flow to SGs = 0 gpm.

Proposed Answer: D Explanation (Optional):

Upon receipt of a LOOP signal, the motor-operated AFW pumps are automatically started by the standby diesel generator load sequencer (Mode II) and run on recirculation until either a steam generator Low-Low level signal, AMSAC, or an SI signal (i.e., AFW flow is necessary) is received. In this situation the LOW - LOW S/G level or AMSAC signal wills open the AFW outside containment isolation valves (OCIVs) and initiate the AFW flow control system. The turbine driven pump will not receive an auto actuation upon a LOOP.

LOT202.28.HO.01, AFW System Student Handout (Rev. 8, page 7, Technical

Reference:

10, 23)

Proposed references to be provided to applicants during examination:

None Learning Objective: (As available)

Question Source: Bank #

Modified Bank # (Note changes or attach parent)

New X Question History: Last NRC Exam Cognitive Level: Memory or Fundamental Knowledge Comprehension or Analysis X 10 CFR Part 55 Content: 55.41 7 55.43 Comments:

KA: 061 AFW System, A3.01 - Ability to monitor automatic operation of the AFW, including:

AFW startup and flows

ES-401 Written Examination Question Worksheet Form ES-401-5 NUREG-1021 Revision-9 Examination Outline Cross-reference: Level RO SRO Tier # 2 Group # 1 K/A # 062.K1.03 Importance Rating 3.5 Proposed Question 46:

Which ONE of the following statements describes the effect of a loss of DC control power to the 4160 VAC bus normal feeder breaker supplying the 4160 VAC bus E1A?

The breaker will:

A. remain in its current position, and can be tripped but not closed from its Control Room Panel.

B. remain in its current position, and cannot be tripped or closed from its Control Room Panel.

C. trip open, and can be closed but not tripped from its Control Room Panel.

D. trip open, and cannot be tripped or closed from its Control Room Panel.

Proposed Answer: B Explanation (Optional):

Class 1E 125 VDC Electrical Distribution System supplies Class 1E 4.16 KV and 480 VAC breaker control power. Without breaker control power, the breaker remains in its current position and cannot be remotely tripped or closed. Thus B is the only correct answer.

LOT201.37 Class 1E 125 VDC Electrical Distribution System Technical

Reference:

Student Handout (Rev. 7, PAGE 27)

Proposed references to be provided to applicants during examination:

None Learning Objective: (As available)

Question Source: Bank # STP - 960 Modified Bank # (Note changes or attach parent)

New Question History: Last NRC Exam Cognitive Level: Memory or Fundamental Knowledge X Comprehension or Analysis 10 CFR Part 55 Content: 55.41 7 55.43 Comments:

KA: 062 AC Electrical Distribution, K1.03 - Knowledge of the physical connections and/or cause-effect relationships between the AC distribution system and the following: DC distribution

ES-401 Written Examination Question Worksheet Form ES-401-5 NUREG-1021 Revision-9 Examination Outline Cross-reference: Level RO SRO Tier # 2 Group # 1 K/A # 063.A3.01 Importance Rating 2.7 Proposed Question 47:

Unit 2 is operating at 100% power with a normal full-power lineup when the 125V DC SYSTEM E2D11 TRBL alarm is received in the Control Room. The operators observe the following indications on CP-003:

  • E2D11 Bus volts: 120 VDC
  • E2D11 Battery Current: minus ( -) 100 amps Based on these indications, which of the below describes what has occurred?

A. E2D11 Battery output breaker has tripped open.

B. The Battery Charger aligned to 125 VDC Bus E2D11 has tripped (de-energized)

C. The Standby Battery Charger for 125 VDC Bus E2D11 has automatically assumed the 125 VDC bus loads.

D. The Inverter/Rectifier associated with 125 VDC Bus E2D11 has tripped (de-energized).

Proposed Answer: B Explanation (Optional):

A. If the Battery output breaker opens there would be no charging or discharging current and 125 VDC bus volts would be normal.

B. Correct answer.

C. The standby charger is normally not in service. There is no automatic feature.

D. The Inverter/Rectifier associated with 125 VDC Bus E2D11 is a load to the Battery and Charger. A loss of this load will not change 125 VDC Bus E2D11, only reduce the amount of current required from the Batter/Battery Charger.

LOT201.37 Class 1E 125 VDC Electrical Distribution System Technical

Reference:

Student Handout (Rev. 7)

Proposed references to be provided to applicants during examination:

None Learning Objective: (As available)

Question Source: Bank #

Modified Bank # (Note changes or attach parent)

New X Question History: Last NRC Exam Cognitive Level: Memory or Fundamental Knowledge Comprehension or Analysis X 10 CFR Part 55 Content: 55.41 7

55.43 Comments:

KA: 063 DC Electrical Distribution System, A3.01 - Ability to monitor automatic operation of the DC electrical system, including: meters, annunciators, dials, recorders, and indicating lights

ES-401 Written Examination Question Worksheet Form ES-401-5 NUREG-1021 Revision-9 Examination Outline Cross-reference: Level RO SRO Tier # 2 Group # 1 K/A # 064.K6.07 Importance Rating 2.7 Proposed Question 48:

Given the following conditions:

  • Unit 1 is at 100% power
  • DG 12 TRBL alarm annunciates Subsequent investigation reveals Starting Air Receiver 13 indicates 242 psig and Starting Air Receiver indicates 85 psig.

According to 0POP02-DG-0002, Emergency Diesel Generator 12 (22) which ONE of the following correctly describes the condition of DG 12?

A. DG 12 is unavailable in the Emergency Mode since starting air pressure <100 psig in one receiver will cause a DG trip signal.

B. DG 12 is operable since starting air pressure is >175 psig in one receiver.

C. DG 12 is unavailable in the Non-Emergency Mode since starting air pressure <

175 psig in one receiver will cause a DG trip signal.

D. DG 12 is NOT operable since starting air pressure is < 100 psig in one receiver.

Proposed Answer: B Explanation (Optional):

A. Incorrect - Air receiver pressure does NOT cause a DG trip signal in either Emergency Mode or Non-Emergency Mode.

B. Correct C. Incorrect - Air receiver pressure does NOT cause a DG trip signal in either Emergency Mode or Non-Emergency Mode.

D. Incorrect - 0POP02-DG-002, Section 4.36: WHEN either of the DG Starting Air Receivers contain LESS THAN 100 psig, THEN the "DG AVAILABLE FOR EMERGENCY" white light will be OFF {ZLP104} and Annunciator Lampbox 104, Window F-3, "DG BYPASSED OR INOPERABLE" will be illuminated. This condition DOES NOT render the DG inoperable.

Technical

Reference:

0POP02-DG-0002 (Section 4.36)

Proposed references to be provided to applicants during examination:

None Learning Objective: (As available)

Question Source: Bank #

Modified Bank # STP - 952 (Note changes or attach parent)

New Question History: Last NRC Exam

Cognitive Level: Memory or Fundamental Knowledge Comprehension or Analysis X 10 CFR Part 55 Content: 55.41 8 55.43 Comments:

KA: 064 Emergency Diesel Generator System, K6.07 - Knowledge of the effect of a loss or malfunction of the following will have on the EDG system: Air receivers

ES-401 Written Examination Question Worksheet Form ES-401-5 NUREG-1021 Revision-9 Examination Outline Cross-reference: Level RO SRO Tier # 2 Group # 1 K/A # 064.A2.11 Importance Rating 2.6 Proposed Question 49:

ESF DG #13 has been operating at full load for a period of time to satisfy surveillance requirements. 0POP02-DG-0003, Emergency Diesel Generator 13 (23), contains recommended Unloading Rates. Based on the Notes/Precautions of 0POP02-DG-0003, these rates are based on which of the following?

A. Optimum engine life and reliability B. Manfacturers warranty requirements C. Prevent going below minimum load limits D. Maintaining load stability on 4160 V ESF Bus E1C Proposed Answer: A Explanation (Optional):

Section 4, Notes and Precautions (0POP02-DG-0003), Step 4.45: In order to ensure optimum engine life and reliability, it is important to operate an engine, whenever possible, in a manner that allows for gradual temperature changes and stabilization periods. Thus A is the only correct answer.

Technical

Reference:

0POP02-DG-0003, Emergency Diesel Generator 13 (23) (Rev 40)

Proposed references to be provided to applicants during examination:

None Learning Objective: (As available)

Question Source: Bank #

Modified Bank # (Note changes or attach parent)

New X Question History: Last NRC Exam Cognitive Level: Memory or Fundamental Knowledge X Comprehension or Analysis 10 CFR Part 55 Content: 55.41 10 55.43 Comments:

KA: 064 Emergency Diesel Generator System, A2.11 - Ability to (a) predict the impacts of the following malfunctions or operations on the EDG system; and (b) based on those predictions, use procedures to correct, control, or mitigate the consequences of those malfunctions or operations: Conditions (minimum load) required for unloading an EDG

ES-401 Written Examination Question Worksheet Form ES-401-5 NUREG-1021 Revision-9 Examination Outline Cross-reference: Level RO SRO Tier # 2 Group # 1 K/A # 073.K4.01 Importance Rating 4.0 Proposed Question 50:

What is the control function of Reactor Containment Building Ventilation System effluent radiation monitors RT-8012 & 8013?

A. Sends a signal to the SSPS for Containment Ventilation Isolation (CVI).

B. Sends a signal to the GWPS shutdown circuitry to close the intake and exhaust valves.

C. Sends a signal to initiate Control Room/EAB emergency ventilation.

D. Sends a signal to initiate FHB exhaust filtration.

Proposed Answer: A Explanation (Optional):

A. Correct - Containment Building Ventilation System RT-8012 & 8013 - High radiation in the RCB Purge System Exhaust sends a signal to the Solid State Protection System (SSPS) for Containment Ventilation Isolation (CVI). (Normal and supplementary purge)

B. Incorrect - Gaseous Waste Processing System (GWPS) - RT-8032 High radiation as measured at the GWPS discharge or a monitor failure condition results in the shutdown of the GWPS. The High Rad or Monitor Failure sends a signal to the GWPS shutdown circuitry to close the discharge valve, the inlet valve , the BRS vent and secure the Bellows Compressor.

C. Incorrect - Electrical Auxiliary Building and Control Room Envelope (HVAC) - RT-8033 &

8034 High radiation level at the EAB air intake initiates Control Room/EAB emergency ventilation.

D. Incorrect - Fuel Handling Building HVAC System -RT-8035 & 8036 High radiation at the exhaust initiates FHB exhaust filtration.

Technical

Reference:

LOT202.41.HO01.REV13 (PAGE 26 OF 44)

Proposed references to be provided to applicants during examination:

None Learning Objective: (As available)

Question Source: Bank #

Modified Bank # (Note changes or attach parent)

New X Question History: Last NRC Exam Cognitive Level: Memory or Fundamental Knowledge X Comprehension or Analysis 10 CFR Part 55 Content: 55.41 7 55.43

Comments:

Original K&A replaced (073.K4.02) - CVCS PRM does not auto isolate letdown at STP KA: 073 Process Radiation Monitoring (PRM) System, K4.01 - Knowledge of PRM system design feature(s) and/or interlocks which provide for the following: Release termination when radiation exceeds setpoint

ES-401 Written Examination Question Worksheet Form ES-401-5 NUREG-1021 Revision-9 Examination Outline Cross-reference: Level RO SRO Tier # 2 Group # 1 K/A # 076.K2.08 Importance Rating 3.1*

Proposed Question 51:

Given the following for Unit 2:

  • ECW Train 2B is in service
  • An SI signal is actuated
  • The BLWDN ISOL FV-6935 F/CLOSE light is lit on A Train ECW Status Monitoring Panel What operator action can be taken to close the ECW Blowdown Valve?

A. Locally verify the ECW Train 2B Blowdown Valve is shut as indicated by the lamp on the ESF Status Monitoring Panel.

B. Operate the ECW Train 2B Blowdown Valve Control switch located at CP-002 from the AUTO position to the CLOSE position to force the valve closed.

C. Locally operate the ECW Train 2A Blowdown Valve from the OPEN position to the CLOSED position.

D. Open the disconnect switch for the ECW Blowdown Valve in the Train A Auxiliary Relay Panel in the Train A 4160V Switchgear Room.

Proposed Answer: D Explanation (Optional):

ECW blowdown valve control switches are kept in the "CLOSE" position by procedure, except ECW Train 2A. Its blowdown valve is normally open to allow pumping the ECW Sump to the ECP. The valve will open even if the ECW Pump is not running when the control switch is in the "AUTO" position. If the ECW Blowdown Valve does not automatically close on an SI signal, the ESF Status Monitoring Panel lamp for "ECW TRN BLOWDOWN VALVE" will light.

A fused power disconnect switch is provided to remove control power and force the valve to its fail closed position. The disconnect switch is in an Auxiliary Relay Panel (ERR-119A, 121B, 122C) in the associated 4160 volt Switchgear Room.

NLO100.29.HO.1, Essential Cooling Water (ECW) and Ventilation Technical

Reference:

System Student Handout (Rev. 9, page 14 of 25);

0POP02-EW-0001, Essential Cooling Water Operations (Rev 36)

Proposed references to be provided to applicants during examination:

None Learning Objective: (As available)

Question Source: Bank #

Modified Bank # (Note changes or attach parent)

New X

Question History: Last NRC Exam Cognitive Level: Memory or Fundamental Knowledge Comprehension or Analysis X 10 CFR Part 55 Content: 55.41 7 55.43 Comments:

KA: 076 Service Water System, K2.08 - Knowledge of bus power supplies to the following:

ESF-actuated MOVs

ES-401 Written Examination Question Worksheet Form ES-401-5 NUREG-1021 Revision-9 Examination Outline Cross-reference: Level RO SRO Tier # 2 Group # 1 K/A # 076.A4.01 Importance Rating 2.9 Proposed Question 52:

Which ONE of the following is NOT an auto start signal for an Essential Cooling Water pump?

A. SI actuation signal (Mode 1).

B. Auto start of the train ESF DG.

C. Low CCW header pressure < 76 psig.

D. ECW pressure in the other two ECW Trains < 30 psig.

Proposed Answer: B Explanation (Optional):

A. Incorrect. An ESF Sequencer start signal will start the ECW pumps after a time delay (Modes I, II, and III).

B. Correct. This is NOT an auto start signal for an ECW pump.

C and D. Incorrect. With the control switches for a non-running Train in the "AUTO" and "CONT RM" positions and the associated ECW/CCW Train Selector Switch in the "STANDBY" position, the ECW/CCW pumps in that Train will automatically start and annunciator "CCW TRAIN AUTO START" will be actuated on the train's ESF Control Panel after a 15 second time delay if either of two conditions occur: ECW pressure in the other two ECW Trains goes below 30 PSIG, or CCW common header pressure goes below 76 PSIG.

NLO100.29.HO.1, Essential Cooling Water (ECW) and Ventilation Technical

Reference:

System Student Handout (Rev. 9, pages 10 and 11 of 25)

Proposed references to be provided to applicants during examination:

None Learning Objective: (As available)

Question Source: Bank #

Modified Bank # (Note changes or attach parent)

New X Question History: Last NRC Exam Cognitive Level: Memory or Fundamental Knowledge X Comprehension or Analysis 10 CFR Part 55 Content: 55.41 7 55.43 Comments:

KA: 076 Service Water System, A4.01 - Ability to manually operate and/or monitor in the control room: ECW pumps

ES-401 Written Examination Question Worksheet Form ES-401-5 NUREG-1021 Revision-9 Examination Outline Cross-reference: Level RO SRO Tier # 2 Group # 1 K/A # 078.K1.02 Importance Rating 2.7*

Proposed Question 53:

Which ONE of the following accurately reflects the correct sequence of events as Instrument Air pressure drops from the normal operating value?

A. Air Compressor 14 (24) starts/loads, Service Air Isolation Valve PV-9785 closes, Instrument Air to Yard Valve PV-8568 closes, Instrument Air Dryer Bypass Valve PV-9983 opens.

B. Instrument Air to Yard Valve PV-8568 closes, IA Compressor 14 (24) starts/loads, Instrument Air Dryer Bypass Valve PV-9983 opens, Service Air Isolation Valve PV-9785 closes.

C. Air Compressor 14 (24) starts/loads, Instrument Air Dryer Bypass Valve PV-9983 opens, Service Air Isolation Valve PV-9785 closes, Instrument Air to Yard Valve PV-8568 closes.

D. Service Air Isolation Valve PV-9785 closes, IA Compressor 14 (24) starts/loads, Instrument Air Dryer Bypass Valve PV-9983 opens, Service Air Isolation Valve PV-9785 closes Proposed Answer: A Explanation (Optional):

(Remote Control) LOAD IDLE First Compressor 117 psi 127 psi Second Compressor 115 psi 125 psi Third Compressor 113 psi 123 psi 100 psig - SAS Isolation Valve closes 90 psig - IA to Yard isolates 90 psig - IAS HDR PRESS LO alarm 88 psig - LP FRVs drift closed 85 psig - SAS HDR PRESS LO alarm 80 psig - Letdown Orifice HDR Isolation Valves drift closed 80 psig - IA Dryer Bypass opens 67 psig - Main FRVs drift closed 60 psig - manual reactor trip NLO200.15.HO.1, Service and Instrument Air Student Handout (Rev. 7, page 12 of 23);

Technical

Reference:

0POP04-IA-0001, Loss Of Instrument Air (Rev 11) 0POP02-IA-0003, Instrument Air System Operation (Rev 6)

Proposed references to be provided to applicants during examination:

None

Learning Objective: (As available)

Question Source: Bank #

Modified Bank # (Note changes or attach parent)

New X Question History: Last NRC Exam Cognitive Level: Memory or Fundamental Knowledge X Comprehension or Analysis 10 CFR Part 55 Content: 55.41 7 55.43 Comments:

KA: 078 Instrument Air, K1.02 - Knowledge of the physical connections and/or cause-effect relationships between the IAS and the following systems: Service Air

ES-401 Written Examination Question Worksheet Form ES-401-5 NUREG-1021 Revision-9 Examination Outline Cross-reference: Level RO SRO Tier # 2 Group # 1 K/A # 103.K3.02 Importance Rating 3.8 Proposed Question 54:

Which ONE of the following conditions concerning the Personnel Air Lock would exceed a Limiting Condition for Operation and require entering a Tech Spec Action Statement?

A. The outer and inner doors are opened simultaneously for a transit entry into containment while in MODE 4.

B. One air lock door fails acceptance test criteria while the plant is in MODE 6.

C. Welding cables are laid through both airlock doors while the plant is in MODE 5.

D. The outer door is opened with inner door shut during a transit entry into containment while in MODE 3.

Proposed Answer: A Explanation (Optional):

3.6.1.3 Each containment air lock shall be OPERABLE with:

b. Both doors closed except when the air lock is being used for normal transit entry and exit through the containment, then at least one air lock door shall be closed.

APPLICABILITY: MODES 1, 2, 3, and 4.

Technical

Reference:

Tech Spec 3.6.1.3 Proposed references to be provided to applicants during examination:

None Learning Objective: (As available)

Question Source: Bank # STP - 1189 Modified Bank # (Note changes or attach parent)

New Question History: Last NRC Exam Cognitive Level: Memory or Fundamental Knowledge X Comprehension or Analysis 10 CFR Part 55 Content: 55.41 9 55.43 Comments:

KA: 103 Containment System, K3.02 - Knowledge of the effect that a loss or malfunction of the containment system will have on the following: Loss of containment integrity under normal operations

ES-401 Written Examination Question Worksheet Form ES-401-5 NUREG-1021 Revision-9 Examination Outline Cross-reference: Level RO SRO Tier # 2 Group # 1 K/A # 103.A2.04 Importance Rating 3.5*

Proposed Question 55:

During fuel handling operations, a Containment Evacuation Alarm is sounded. Which ONE of the following identifies actions required to isolate containment as listed in 0POP04-FH-0001, Fuel Handling Accident?

A. PA announcement to evacuate the RCB; check at least one door in each containment air lock clear of all obstructions and capable of being closed B. check FHB HVAC operating in Emergency Mode; close at least one door in each containment air lock C. check both doors closed in each containment air lock, check equipment hatch in place and secured D. check automatic containment ventilation isolation; close at least one door in each containment air lock Proposed Answer: D Explanation (Optional):

A - Incorrect. At least one door is required to be closed in each air lock, not just capable of being closed B - Incorrect. FHB HVAC operating in Emergency Mode is action for Fuel Handling Accident in FHB C - Incorrect. Only one door in each air lock is required to be closed D - Correct.

Technical

Reference:

0POP04-FH-0001, Fuel Handling Accident (Rev 7)

Proposed references to be provided to applicants during examination:

None Learning Objective: (As available)

Question Source: Bank #

Modified Bank # (Note changes or attach parent)

New X Question History: Last NRC Exam Cognitive Level: Memory or Fundamental Knowledge X Comprehension or Analysis 10 CFR Part 55 Content: 55.41 10 55.43 Comments:

KA: 103 Containment System, A2.04 - Ability to (a) predict the impacts of the following

malfunctions or operations on the containment system; and (b) based on those predictions, use procedures to correct, control, or mitigate the consequences of those malfunctions or operations: Containment evacuation (including recognition of the alarm)

ES-401 Written Examination Question Worksheet Form ES-401-5 NUREG-1021 Revision-9 Examination Outline Cross-reference: Level RO SRO Tier # 2 Group # 2 K/A # 001.K4.02 Importance Rating 3.8 Proposed Question 56:

To recover a dropped Control Bank B rod, the Rod Bank Selector Switch must be in the Control Bank B position to allow:

A. the dropped rod to move without moving rods in other Control Banks through the Lift Coil Disconnect Switches.

B. rods in other Control Banks to move (when commanded) through the Bank Overlap Unit so they maintain their overlap alignment.

C. the dropped rod to move without moving rods in other Control Banks through the Bank Overlap Unit.

D. rods in the other Control Banks to move (when commanded) through the Reactor Control Unit so they maintain their overlap alignment.

Proposed Answer: C Explanation (Optional):

The bank overlap feature is disabled when the Rod Bank Selector Switch is out of either AUTO or MANUAL so only the rod bank selected will move when commanded. This makes C correct and B and D incorrect. A is incorrect since Lift Coil Disconnect Switches do not function as a result of the position of the Rod Bank Selector Switch.

Technical

Reference:

LOT201.18.HO.01, Rod Control System (Rev. 9)

Proposed references to be provided to applicants during examination:

None Learning Objective: (As available)

Question Source: Bank #

Modified Bank # (Note changes or attach parent)

New X Question History: Last NRC Exam Cognitive Level: Memory or Fundamental Knowledge X Comprehension or Analysis 10 CFR Part 55 Content: 55.41 2 55.43 Comments:

KA: 001 Control Rod Drive System, K4.02 - Knowledge of the CRDS design feature(s) and/or interlock(s) which provide for the following: Control rod mode select control (movement control)

ES-401 Written Examination Question Worksheet Form ES-401-5 NUREG-1021 Revision-9 Examination Outline Cross-reference: Level RO SRO Tier # 2 Group # 2 K/A # 014.A1.04 Importance Rating 3.5 Proposed Question 57:

Unit 1 is operating at full power when a transient occurs that necessitates a rapid power reduction. During the power reduction the BANK INSRT LO-LO alarm is received. Which ONE of the below correctly describes the operator action that should be taken and the effect on core power distribution?

A. Commence boration of the RCS. Control rod position is at the Rod Insertion Limit.

Both axial and radial power distribution are impacted by the current control rod positions.

B. Place rods in MANUAL and withdraw rods to clear the alarm. Control rod position is at the Rod Insertion Limit. Both axial and radial power distribution are impacted by the current control rod positions.

C. Commence boration of the RCS. Control rod position is 10 steps above the Rod Insertion Limit. Only axial power distribution is impacted by the current control rod positions.

D. Place rods in MANUAL. Control rod position is 10 steps above the Rod Insertion Limit. Only axial power distribution is impacted by the current control rod positions.

Proposed Answer: A Explanation (Optional):

Annunciator Response Instructions contain immediate actions to COMPARE rod bank positions on DRPI with Rod Insertion Limits. IF any RCCA bank is positioned below the Rod Insertion Limits for the current reactor power, THEN GO TO 0POP04-CV-0003, Emergency Boration. BANK INSRT LO-LO alarm is annunciated when rods are at the Rod Insertion Limit. BANK INSRT LO alarm is annunciated when rods are 10 steps above the Rod Insertion Limit. Rod height affects both axial and radial power distribution. Thus, A is the only correct answer.

LOT201.19, Rod Position Indicating System Student Handout (Rev.

11);

Technical

Reference:

0POP09-AN-5M03, Annunciator Lampbox 5M03 Response Instructions (Rev 23), Window E4 Proposed references to be provided to applicants during examination:

None Learning Objective: (As available)

Question Source: Bank #

Modified Bank # (Note changes or attach parent)

New X Question History: Last NRC Exam

Cognitive Level: Memory or Fundamental Knowledge Comprehension or Analysis X 10 CFR Part 55 Content: 55.41 5 55.43 Comments:

KA: 014 Rod Position Indication System, A1.04 - Ability to predict and/or monitor changes in parameters (to prevent exceeding design limits) associated with operating the RPIS controls, including: Axial and radial power distribution

ES-401 Written Examination Question Worksheet Form ES-401-5 NUREG-1021 Revision-9 Examination Outline Cross-reference: Level RO SRO Tier # 2 Group # 2 K/A # 015.K5.15 Importance Rating 3.3 Proposed Question 58:

Unit 1 is at 90% power with Delta-I stable at -2 %. If Control Bank D is inserted while diluting to maintain Tavg on program, the operator would expect which ONE of the following to occur?

A. Delta-I will get LESS NEGATIVE, Xenon will INCREASE in the top of the core.

B. Delta-I will get LESS NEGATIVE, Xenon will DECREASE in the top of the core.

C. Delta-I will get MORE NEGATIVE, Xenon will INCREASE in the top of the core.

D. Delta-I will get MORE NEGATIVE, Xenon will DECREASE in the top of the core.

Proposed Answer: C Explanation (Optional):

Insertion of control rods reduces the flux in the top region of the core thus less leakage of fast neutrons at the upper detectors thus a MORE negative Delta-I. Reduced power in the top region of the core will reduce Xe burnout causing Xe concentration to INCREASE.

LOT201.16.HO.01, Excore Nuclear Instrumentation Student Technical

Reference:

Handout (Rev. 12, page 33 of 42)

Proposed references to be provided to applicants during examination:

None Learning Objective: (As available)

Question Source: Bank #

Modified Bank # (Note changes or attach parent)

New X Question History: Last NRC Exam Cognitive Level: Memory or Fundamental Knowledge X Comprehension or Analysis 10 CFR Part 55 Content: 55.41 1, 5 55.43 Comments:

KA: 015 Nuclear Instrumentation System, K5.15 - Knowledge of the operational implications of the following concepts as they apply to the NIS: Effects of xenon on local flux, and factors affecting xenon concentrations

ES-401 Written Examination Question Worksheet Form ES-401-5 NUREG-1021 Revision-9 Examination Outline Cross-reference: Level RO SRO Tier # 2 Group # 2 K/A # 017.A4.02 Importance Rating 3.8 Proposed Question 59:

While responding to an Inadequate Core Cooling situation the operators are directed to reestablish some form of core cooling.

Which ONE of the following statements describes the INITIAL response of the CETs, and the reason for this response, if the operators restarted the safety injection pumps?

A. Increase due to saturated steam being forced out of the core.

B. Increase due to superheated steam being forced out of the core.

C. Decrease due to saturated steam forming a frothy two phase mixture.

D. Decrease due to superheated steam causing injected water to boil forming a frothy two phase mixture.

Proposed Answer: B Explanation (Optional):

A: INCORRECT - Correct response, incorrect basis B: CORRECT - Correct response, correct basis C: INCORRECT - Incorrect response, incorrect basis D: INCORRECT - Incorrect response, incorrect basis 148-00041, WOG background document on Inadequate Core Technical

Reference:

Cooling Rev 1B, page 2 Proposed references to be provided to applicants during examination:

None Learning Objective: T20117 (As available)

Question Source: Bank # STP - 47 Modified Bank # (Note changes or attach parent)

New Question History: Last NRC Exam Cognitive Level: Memory or Fundamental Knowledge Comprehension or Analysis X 10 CFR Part 55 Content: 55.41 2, 10, 14 55.43 Comments:

KA: 017 In-Core Temperature Monitoring System (ITM), A4.02 - Ability to manually operate and/or monitor in the control room: Temperature values used to determine RCS/RCP

operation during inadequate core cooling (i.e., if applicable, average of five highest values)

Note from exam bank: Candidates must analyze the conditions and determine that inadequate core cooling is > 1200 degrees-F and that once SI pumps are started superheated steam is forced out of the core area past the CETs causing temperature indication to initially increase. During the Facility validation of the written exam the SROs who were taking the exam felt that the question was hard but was a very good question.

ES-401 Written Examination Question Worksheet Form ES-401-5 NUREG-1021 Revision-9 Examination Outline Cross-reference: Level RO SRO Tier # 2 Group # 2 K/A # 028.A1.01 Importance Rating 3.4 Proposed Question 60:

Given the following:

  • Unit 1 was at 100% power when it tripped due to a LOCA
  • Hydrogen Recombiner is being placed in service in accordance with 0POP02-CG-0001, Electric Hydrogen Recombiners Which one of the following is NOT an indication that recombination is occurring after having placed the Hydrogen Recombiner in service?

A. Recombiner temperature is above the threshold for hydrogen recombination.

B. Recombiner power consumption is indicated by a KW output level on the associated CP002 wattmeter.

C. Hydrogen concentration is being controlled or reduced by the Recombiner as indicated on the containment hydrogen recorder on CP018.

D. Containment pressure decreases after the Hydrogen Recombiners are placed in service.

Proposed Answer: D Explanation (Optional):

0POP05-EO-EO10 requires a Hydrogen Recombiner to be placed in service if H2 > 0.5 %

IAW 0POP02-CG-0001. Section 6.0 contains a note that identifies three independent means of verifying Recombiner operation (Answers A-C above). Answer D is a correct statement if H2 concentration decreases or remains constant, but it is not an indication of Recombiner operation.

Technical

Reference:

0POP02-CG-0001 Proposed references to be provided to applicants during examination:

None Learning Objective: (As available)

Question Source: Bank #

Modified Bank # CPSES (Note changes or attach parent)

New Question History: Last NRC Exam Cognitive Level: Memory or Fundamental Knowledge Comprehension or Analysis X

10 CFR Part 55 Content: 55.41 10 55.43 Comments:

KA: 028 Hydrogen Recombiner and Purge Control System (HRPS), A1.01 - Ability to predict and/or monitor changes in parameter (to prevent exceeding design limits) associated with operating the HRPS controls including; Hydrogen concentration CPSES Exam Bank, Objective: SYS.CY1.OB900. Modified stem and changed distractors by having applicant identify the INCORRECT means to verify recombiner operation.

ES-401 Written Examination Question Worksheet Form ES-401-5 NUREG-1021 Revision-9 Examination Outline Cross-reference: Level RO SRO Tier # 2 Group # 2 K/A # 029.K3.01 Importance Rating 2.9 Proposed Question 61:

Unit 2 Containment pressure is 0.4 psig and the crew has decided to conduct a containment purge. The following conditions exist:

  • Mode 4
  • Supplementary Purge Supply Fan 21A is out of service
  • Supplementary Purge Inlet and Outlet Dampers are opened
  • Supplementary Purge Exhaust Fan is started
  • Supplementary Purge Supply Fan 21B fails to start.

Which ONE of the following describes the effect the given conditions have on Containment pressure?

A. Will remain at 0.4 psig until two of three trains of RCFCs can be started.

B. Will eventually equalize without either Supplementary Purge Supply Fan running.

C. Will remain at 0.4 psig until one of the two Supplementary Purge Supply Fans can be started.

D. Will become negative without either Supplementary Purge Supply Fan running.

Proposed Answer: D Explanation (Optional):

A. Incorrect - containment pressure will not remain at .4 psig regardless of RCFC's.

B. Incorrect - containment pressure will not equilize given current conditions C. Incorrect - containment pressure will not remain at .4 psig D. Correct - running an Exhaust Fan without a running Supply Fan will cause a negative pressure in the RCB 0POP02-HC-0003, Supplementary Containment Purge Technical

Reference:

(Rev 17, Section 6.0)

Proposed references to be provided to applicants during examination:

None Learning Objective: (As available)

Question Source: Bank #

Modified Bank # (Note changes or attach parent)

New X Question History: Last NRC Exam Cognitive Level: Memory or Fundamental Knowledge Comprehension or Analysis X

10 CFR Part 55 Content: 55.41 7 55.43 Comments:

KA: 029 Containment Purge System, K3.01 - Knowledge of the effect that a loss or malfunction of the Containment Purge System will have on the following: Containment parameters

ES-401 Written Examination Question Worksheet Form ES-401-5 NUREG-1021 Revision-9 Examination Outline Cross-reference: Level RO SRO Tier # 2 Group # 2 K/A # 041.G2.1.2 Importance Rating 3.0 Proposed Question 62:

Which one of the following describes the operator action required to cool the unit below the P-12 setpoint?

Place the steam dumps in the:

A. Tave (Turbine Trip) Mode, then momentarily take the Mode Selector switch to the RESET position.

B. Tave (Turbine Trip) Mode, then momentarily take both Interlock Select switches to the BYPASS position.

C. Steam Pressure Mode, then momentarily take the Mode Selector switch to the RESET position.

D. Steam Pressure Mode, then momentarily take both Interlock Select switches to the BYPASS position.

Proposed Answer: D Explanation (Optional):

Per Tech Ref, steam pressure mode is selected during low power operation, below 15 percent or upon establishing a hot standby condition. The operator will maintain no load steam header pressure by selecting STEAM PRESSURE on the Steam Dump Mode Selector Switch. For cooldown below the P-12 setpoint (Tave < 563F), cooldown may continue by momentarily cycling the Steam Dump Interlock Switches to the BYPASS Interlock position.

This allows operation of the Bank No. 1 dump valves only.

Technical

Reference:

LOT202.09.HO.01, Steam Dump System (Rev. 11, page 24 of 32)

Proposed references to be provided to applicants during examination:

None Learning Objective: (As available)

Question Source: Bank # CPSES Modified Bank # (Note changes or attach parent)

New Question History: Last NRC Exam Cognitive Level: Memory or Fundamental Knowledge Comprehension or Analysis X 10 CFR Part 55 Content: 55.41 7 55.43 Comments:

KA: 041 Steam Dump System and Turbine Bypass Control, G2.1.2 - Knowledge of operator responsibilities during all modes of plant operation.

From CPSES Exam Bank, KSA: 041.020.K4.09 (3.0, 3.3)

ES-401 Written Examination Question Worksheet Form ES-401-5 NUREG-1021 Revision-9 Examination Outline Cross-reference: Level RO SRO Tier # 2 Group # 2 K/A # 072.A3.01 Importance Rating 2.9*

Proposed Question 63:

Given the following conditions on Unit 1:

  • 100% power
  • RCB Supplementary Purge System is in operation
  • Containment High Range Radiation Monitor (RT-8050) has HIGH alarm Which ONE of the following identifies the expected alignment of RCB HVAC five (5) minutes after the high radiation alarm?

A. Normal Purge Supply Fans - RUNNING Normal Purge Exhaust Isolation Valves - OPEN Normal Purge Exhaust Fans - RUNNING Normal Purge Supply Isolation Valves - OPEN B. Normal Purge Supply Fans - OFF Normal Purge Exhaust Isolation Valves - OPEN Normal Purge Exhaust Fans - OFF Normal Purge Supply Isolation Valves - OPEN C. Supplementary Purge Supply Fans - RUNNING Supplementary Purge Exhaust Isolation Valves - OPEN Supplementary Purge Exhaust Fans - RUNNING Supplementary Purge Supply Isolation Valves - OPEN D. Supplementary Purge Supply Fans - OFF Supplementary Purge Exhaust Isolation Valves - CLOSED Supplementary Purge Exhaust Fans - OFF Supplementary Purge Supply Isolation Valves - CLOSED Proposed Answer: C Explanation (Optional):

Applicant must recognize that Normal Containment Purge is NOT operated in Modes 1-4, and must recognize that Containment High Rad Monitors (RT-8050 and RT-8051) do NOT interface with the containment ventilation system. Any alarming condition will NOT cause a CVI, therefore containment ventilation valves will NOT change status.

LOT202.33.HO.01, RCB HVAC Student Handout (Rev 6);

Technical

Reference:

0POP04-RA-0001, Rad Monitoring Sys Alarm Response Rev 16);

0POP02-HC-0002, Normal Containment Purge (Rev 10);

0POP02-HC-0003, Supplementary Containment Purge (Rev 17)

Proposed references to be provided to applicants during examination:

None Learning Objective: (As available)

Question Source: Bank #

Modified Bank # (Note changes or attach parent)

New X Question History: Last NRC Exam Cognitive Level: Memory or Fundamental Knowledge Comprehension or Analysis X 10 CFR Part 55 Content: 55.41 11 55.43 Comments:

Original K&A replaced (072.K4.03) - ARM system has no interface with plant ventilation KA: 072 Area Radiation Monitoring System, A3.01 - Ability to monitor automatic operation of the ARM system, including: Changes in ventilation alignment

ES-401 Written Examination Question Worksheet Form ES-401-5 NUREG-1021 Revision-9 Examination Outline Cross-reference: Level RO SRO Tier # 2 Group # 2 K/A # 075.A2.02 Importance Rating 2.5 Proposed Question 64:

Unit 1 is operating at 100% power. Three Circulating Water Pumps are operating when a CWP TRIP/FAIL START alarm is received. Indications show CWP # 11 has tripped for an unknown reason.

According to 0POP04-CW-0001, Loss of Circulating Water Flow, which ONE of the below correctly describes the Control Room actions to take (sequence is unimportant)?

A. Trip the reactor, trip the turbine, go to 0POP05-EO-EO00, Reactor Trip or Safety Injection.

B. Ensure CWP #11 discharge valves closes, start the standby CWP and commence a load reduction to maintain condenser vacuum.

C. Close CWP # 11 discharge valve using its control switch on CP-009, start the standby CWP and commence a load reduction to maintain condenser vacuum.

D. Trip the turbine, ensure CWP #11 discharge valves closes, and go to 0POP04-TM-0001, Turbine Load Rejection.

Proposed Answer: B Explanation (Optional):

A. Incorrect. Trip not required B. Corrrect.

C. Incorrect. There is no switch for the CWP discharge valves in the Control Room.

D. Incorrect. Tripping the turbine at this power will cause a reactor trip. Crew would have to go to the reactor trip procedure.

Technical

Reference:

0POP04-CW-0001, Loss of Circulating Water Flow (Rev 1)

Proposed references to be provided to applicants during examination:

None Learning Objective: (As available)

Question Source: Bank #

Modified Bank # (Note changes or attach parent)

New X Question History: Last NRC Exam Cognitive Level: Memory or Fundamental Knowledge X Comprehension or Analysis 10 CFR Part 55 Content: 55.41 10 55.43

Comments:

KA: 075 Circulating Water System, A2.02 - Ability to (a) predict the impacts of the following malfunctions or operations on the circulating water system; and (b) based on those predictions, use procedures to correct, control, or mitigate the consequences of those malfunctions or operations.

ES-401 Written Examination Question Worksheet Form ES-401-5 NUREG-1021 Revision-9 Examination Outline Cross-reference: Level RO SRO Tier # 2 Group # 2 K/A # 079.A4.01 Importance Rating 2.7 Proposed Question 65:

Unit 2 is operating at 85% power when SAS ISOL VLV CLOSE annuciates. The operater notes the following header pressures:

The operator notes the following header pressures:

  • IA header pressure = 90 psig and decreasing
  • SA header pressure = 100 psig and increasing Given that all systems operate as designed, which of the following describes the status of the IA and SA systems?

A. A significant leak in the IAS has occurred, and automatic closure of Service Air Isolation Valve (PV-9785) has successfully isolated the SAS.

B. A significant leak in the SAS has occurred but automatic starting of all air compressors is maintaining SAS header pressure.

C. A minor leak in either the IAS or SAS has occurred, and automatic closure of Service Air Isolation Valve (PV-9785) has successfully isolated the SAS.

D. The size or location of the leak cannot be determined from given information.

Proposed Answer: A Explanation (Optional):

Candidate must analyze the change in IAS/SAS header pressures and combine with the knowledge that SAS Isolation Valve automatically closes at 100 psig, IAS HDR PRESS LO annuciates at 90 psig, and all four air compressors are running at 113 psig IA pressure.

Since it was given that all systems operate as designed, the SAS Isolation Valve closed at 100 psig (indicated by an increasing SAS header pressure). IA header pressure is continuing to drop even with air receivers and all four compressors running, thus the leak is significant (not minor).

0POP09-AN-08M3, Annunciator Lampbox 1(2)-08M-3 Response Technical

Reference:

Instructions (Rev 31);

0POP04-IA-0001, Loss Of Instrument Air (Rev 11)

Proposed references to be provided to applicants during examination:

None Learning Objective: (As available)

Question Source: Bank #

Modified Bank # (Note changes or attach parent)

New X Question History: Last NRC Exam Cognitive Level: Memory or Fundamental Knowledge Comprehension or Analysis X 10 CFR Part 55 Content: 55.41 7 55.43 Comments:

KA: 079 Station Air System (SAS), A4.01 - Ability to manually operate and/or monitor in the control room: Cross-tie valves is IAS

ES-401 Written Examination Question Worksheet Form ES-401-5 NUREG-1021 Revision-9 Examination Outline Cross-reference: Level RO SRO Tier # 3 Group # 1 K/A # G2.1.21 Importance Rating 3.1 Proposed Question 66:

Which ONE of the following methods is an approved way to assure that a working copy of a procedure is current?

A. As long as the procedure is turned over during a job in progress, it is valid until job completion.

B. The procedure is verified against a Level 1 Station Controlled hardcopy procedure.

C. Verify the working copy is the same revision and contains the same Field Changes as the last completed copy of the procedure.

D. The stamp on the cover sheet has been initialed less than 48 hours5.555556e-4 days <br />0.0133 hours <br />7.936508e-5 weeks <br />1.8264e-5 months <br /> ago.

Proposed Answer: B Explanation (Optional):

Working copies or controlled copies of PROCEDURES other than from a Level 1 Station are verified to be current revision with all effective amendments included PRIOR TO USE by:

  • Inquiry in Oracle RMS/ECM or any Level 1 computer-database. (preferred)

OR

  • Comparison to a Level 1 Station Controlled hardcopy PROCEDURE.

OR

  • Review of daily listing of PROCEDURE changes for continual use operational PROCEDURES.

OR

  • Inquiry to Document Control.

OR

  • Cognizant Managers Signature verified on new revision.

Therefore, B is the only correct answer.

0PGP03-ZA-0010, Performing and Verifying Station Activities Technical

Reference:

(Rev. 26, page 7 of 29)

Proposed references to be provided to applicants during examination:

None Learning Objective: (As available)

Question Source: Bank # STP - 165 Modified Bank # (Note changes or attach parent)

New Question History: Last NRC Exam Cognitive Level: Memory or Fundamental Knowledge X

Comprehension or Analysis 10 CFR Part 55 Content: 55.41 10 55.43 Comments:

KA: G2.1.21 - Ability to obtain and verify controlled procedure copy.

ES-401 Written Examination Question Worksheet Form ES-401-5 NUREG-1021 Revision-9 Examination Outline Cross-reference: Level RO SRO Tier # 3 Group # 1 K/A # G2.1.14 Importance Rating 2.5 Proposed Question 67:

In accordance with Conduct of Operations guidance for use of the Public Address (PA) System, which one of the below plant/system conditions should NOT be announced using the PA?

A. Entering a mid-loop condition B. Starting a Reactor Coolant Pump (RCP)

C. Entering Mode 3 D. Shifting a Battery Charger lineup Proposed Answer: D Explanation (Optional):

The paging system should be used to update plant personnel of the status of abnormal or emergency conditions, notification of change in plant status, or major plant events or evolutions in progress or anticipated.

Technical

Reference:

Conduct of Operations Chapter 3, Section 3.2.3.1 Proposed references to be provided to applicants during examination:

None Learning Objective: (As available)

Question Source: Bank #

Modified Bank # (Note changes or attach parent)

New X Question History: Last NRC Exam Cognitive Level: Memory or Fundamental Knowledge X Comprehension or Analysis 10 CFR Part 55 Content: 55.41 10 55.43 Comments:

KA: G2.1.14 - Knowledge of system status criteria which require the notification of plant personnel.

ES-401 Written Examination Question Worksheet Form ES-401-5 NUREG-1021 Revision-9 Examination Outline Cross-reference: Level RO SRO Tier # 3 Group # 2 K/A # G2.2.2 Importance Rating 4.0 Proposed Question 68:

Which ONE of the following is the correct sequence for paralleling the main generator to the grid?

A. 1) Adjust voltage until the INCOMING VOLTS is slightly higher than the RUNNING VOLTS

2) Place the GEN BKR SYNC SW in the ON position
3) Adjust Main Turbine speed so the SYNCHROSCOPE needle is rotating slowly in the clockwise direction
4) Close the Main Generator Exciter Field Breaker
5) Close the Main Generator Breaker B. 1) Close the Main Generator Exciter Field Breaker
2) Adjust Main Turbine speed so the SYNCHROSCOPE needle is rotating slowly in the clockwise direction
3) Adjust voltage until the INCOMING VOLTS is slightly higher than the RUNNING VOLTS
4) Place the GEN BKR SYNC SW in the ON position
5) Close the Main Generator Breaker C. 1) Place the GEN BKR SYNC SW in the ON position
2) Adjust Main Turbine speed so the SYNCHROSCOPE needle is rotating slowly in the clockwise direction
3) Adjust voltage until the INCOMING VOLTS is slightly higher than the RUNNING VOLTS
4) Close the Main Generator Exciter Field Breaker
5) Close the Main Generator Breaker D. 1) Close the Main Generator Exciter Field Breaker
2) Place the GEN BKR SYNC SW in the ON position
3) Adjust Main Turbine speed so the SYNCHROSCOPE needle is rotating slowly in the clockwise direction
4) Adjust voltage until the INCOMING VOLTS is slightly higher than the RUNNING VOLTS
5) Close the Main Generator Breaker Proposed Answer: D Explanation (Optional):

A: INCORRECT - The field breaker must be closed in order to adjust the generator voltage or get a synchroscope output.

B: INCORRECT - The synch switch must be on in order to get a reading on the incoming and running voltmeters and to get a synchroscope output.

C: CORRECT - Per the plant startup procedure, the field breaker is closed, then the synch switch turned on, then speed and voltage adjusted and finally the output breaker closed.

D: INCORRECT - Voltage cannot be adjusted until the synch switch is turned on and the field breaker closed.

0POP03-ZG-0005, Plant Startup to 100% (Rev. 48, pages 49-52 of Technical

Reference:

114)

Proposed references to be provided to applicants during examination:

None Learning Objective: (As available)

Question Source: Bank # STP - 103 Modified Bank # (Note changes or attach parent)

New Question History: Last NRC Exam Cognitive Level: Memory or Fundamental Knowledge X Comprehension or Analysis 10 CFR Part 55 Content: 55.41 10 55.43 Comments:

KA: G2.2.2 - Ability to manipulate the console controls as required to operate the facility between shutdown and designated power levels.

Noted on the question from the exam bank: Difficulty Justification - The candidate must have a basic understanding of generator theory and the design of the generator controls. By knowing what must be done to parallel a generator and understanding how the system works, the candidate can determine the correct sequence for performing these steps.

ES-401 Written Examination Question Worksheet Form ES-401-5 NUREG-1021 Revision-9 Examination Outline Cross-reference: Level RO SRO Tier # 3 Group # 2 K/A # G2.2.22 Importance Rating 3.4 Proposed Question 69:

Unit 1 is operating at full power with an A Train outage in progress (A HHSI, A LHSI, A CCW components are out of service). All other equipment is operable.

  • B ECW pump trips Which of the below correctly describe the most limiting Tech Spec that should be entered and the reason?

A. Tech Spec 3.0.3 because there are now two ESF DGs inoperable.

B. Tech Spec 3.8.1.1 (A.C. Sources) because there are now two ESF DGs inoperable.

C. Tech Spec 3.0.3 because there are now two trains of ECCS inoperable.

D. Tech Spec 3.5.2 (ECCS Subsystems) because there are now two trains of ECCS inoperable.

Proposed Answer: C Explanation (Optional):

A and B are incorrect because only 1 ESF DG is inoperable. D is incorrect because there is no action statement for 2 or more trains of ECCS being inoperable (thats what makes it a 3.03 condition).

This question requires the candidate to know that for an operable ECCS train there must be an operable RHR Hx. For this Hx to be operable it must have both ECW and CCW. Train A RHR Hx is inoperable because Train A CCW is out of service. Train B RHR Hx is inoperable because Train B ECW is unavailable. Candidate must analyze the given conditions to realize two trains of ECCS are inoperable as well as know what Tech Spec limitations would apply.

TS 3.0.3; Technical

Reference:

TS 3.5.2, ECCS SUBSYSTEMS - Tavg - Greater Than or Equal to 350 F Proposed references to be provided to applicants during examination:

None Learning Objective: (As available)

Question Source: Bank #

Modified Bank # (Note changes or attach parent)

New X Question History: Last NRC Exam

Cognitive Level: Memory or Fundamental Knowledge Comprehension or Analysis X 10 CFR Part 55 Content: 55.41 3 55.43 Comments:

KA: G2.2.22 - Knowledge of limiting conditions for operations and safety limits.

ES-401 Written Examination Question Worksheet Form ES-401-5 NUREG-1021 Revision-9 Examination Outline Cross-reference: Level RO SRO Tier # 3 Group # 2 K/A # G2.2.27 Importance Rating 2.6 Proposed Question 70:

Which ONE of the following correctly identifies the sequence of events for Non-Rapid Refueling Operations following entry into Mode 6?

A. Establish 2 operable trains of RHR, detension reactor head bolts, upcouple control rod drive shafts, fill refueling cavity, remove reactor vessel head, remove upper internals, conduct core alterations B. Uncouple control rod drive shafts, detension reactor head bolts, remove upper internals, conduct core alterations, install upper internals, recouple control rods, tension reactor head bolts C. Establish RCS boron concentration > 2800 ppm, fill refueling cavity, uncouple control rod drive shafts, remove vessel head, refuel core, install reactor head, recouple control rods D. Uncouple control rod drive shafts, remove reactor vessel head, remove upper internals, refuel core, install reactor upper internals, recouple control rods, tension reactor head bolts Proposed Answer: D Explanation (Optional):

A. INCORRECT - 2 trains RHR operable is a requirement to enter Mode 6 B. INCORRECT - Vessel head bolts are detensioned prior to uncoupling control rod drive shafts (tensioning reactor head bolts transitions plant to Mode 5)

C. INCORRECT - Boron concentration > 2800 ppm is a requirement to enter Mode 6 and control rod drive shafts are uncoupled prior to vessel head removal D. CORRECT - Stem states "after entry into Mode 6" so reactor head bolts are less than fully tensioned (tensioning reactor head bolts transitions plant to Mode 5).

Technical

Reference:

0POP03-ZG-0010, Refueling Operations (Rev. 39)

Proposed references to be provided to applicants during examination:

None Learning Objective: (As available)

Question Source: Bank #

Modified Bank # (Note changes or attach parent)

New X Question History: Last NRC Exam Cognitive Level: Memory or Fundamental Knowledge X Comprehension or Analysis

10 CFR Part 55 Content: 55.41 10 55.43 Comments:

KA: G2.2.27 - Knowledge of the refueling process

ES-401 Written Examination Question Worksheet Form ES-401-5 NUREG-1021 Revision-9 Examination Outline Cross-reference: Level RO SRO Tier # 3 Group # 3 K/A # G2.3.9 Importance Rating 2.5 Proposed Question 71:

Given the following:

  • During a mid-cycle reactor startup the Reactor Containment Building (RCB)

CNTMT PRESS HI/LO alarm annunciates with RCB pressure at +0.4 psig

  • T.S. 3.6.1.4 for Containment Systems Internal Pressure is entered.
  • It is determined a Supplementary Purge will be performed to lower RCB pressure
  • Four RCFCs are in operation
  • The Unit Supervisor notes that there is no Form 1, RCB Purge Notification Levels, in the Control Room.

Which ONE of the following describes the action(s) to be taken to lower RCB pressure?

A. The purge may be started provided Chemical Analysis concurs.

B. The purge may be started provided Chemical Analysis continuously samples during the purge.

C. The purge may NOT be started until a valid Form 1, RCB Purge Notification Levels, is in the Control Room.

D. The purge may NOT be started until additional RCFCs are started.

Proposed Answer: A Explanation (Optional):

A: CORRECT: Purge is allowed to meet the 1 hour1.157407e-5 days <br />2.777778e-4 hours <br />1.653439e-6 weeks <br />3.805e-7 months <br /> Tech Spec as long as chemical analysis concurs and a new Form 1 is initiated.

B: INCORRECT: No, a Form 1 must at least be initiated prior to purge operations and a continuous sample is not required.

C: INCORRECT: No, the purge may be started provided a new Form 1 is initiated.

D: INCORRECT: No, the purge may be started provided a new Form 1 is initiated and only 4 RCFC units are required to be in operation.

0POP02-HC-0003, Supplementary Containment Purge (Rev 17);

Technical

Reference:

T.S. 3.6.1.4; Containment Systems - Internal Pressure 0PGP03-ZO-0024 Proposed references to be provided to applicants during examination:

None Learning Objective: (As available)

Question Source: Bank # STP - 445 Modified Bank # (Note changes or attach parent)

New Question History: Last NRC Exam Cognitive Level: Memory or Fundamental Knowledge Comprehension or Analysis X 10 CFR Part 55 Content: 55.41 10 55.43 Comments:

KA: G2.3.9 - Knowledge of the process for performing a containment purge

ES-401 Written Examination Question Worksheet Form ES-401-5 NUREG-1021 Revision-9 Examination Outline Cross-reference: Level RO SRO Tier # 3 Group # 3 K/A # G2.3.2 Importance Rating 2.5 Proposed Question 72:

A point source in the RCA is reading 500 mrem/hr at 1 foot. There are two options being considered to perform a job in the area.

Option #1: Two operators working together are capable of completing the job in 20 minutes at 4 feet from the source.

Option #2: One operator can complete the job in 80 minutes at 8 feet from the source.

Which ONE of the following is the preferred option AND consistent with the goals of the ALARA program?

A. Option #1 - each operators exposure is 10.0-10.5 mrem B. Option #2 - the operators exposure is 5.8-6.2 mrem C. Option #2 - the operators exposure is 10.0-10.5 mrem D. Option #1 - each operators exposure is 5.8-6.2 mrem Proposed Answer: C Explanation (Optional):

A: INCORRECT: Does not support ALARA person-rem limit concept.

B: INCORRECT: Option 2 exposure is 10 mrem.

C: CORRECT: Accurate exposure and supports ALARA person-rem limit concept.

D: INCORRECT: Option 1 exposure is 10 mrem to each operator.

0PGP03-ZR-0052, ALARA Program (Rev 7);

Technical

Reference:

GET002.19.01 Proposed references to be provided to applicants during examination:

None Learning Objective: (As available)

Question Source: Bank # STP - 425 Modified Bank # (Note changes or attach parent)

New Question History: Last NRC Exam Cognitive Level: Memory or Fundamental Knowledge Comprehension or Analysis X

10 CFR Part 55 Content: 55.41 12 55.43 Comments:

Difficulty Justification: The exposure for each operator for the job is the same (10.0-10.5 mrem depending on places carried in the hour multiplier). The ALARA program stresses the need to limit not only individual exposure but also total person-rem. For Option #1 the total exposure to both workers (10 + 10 = 20mrem) would be greater than that of Option #2 (10 mrem). This item requires the candidate to understand the ALARA concept, calculate the exposures and apply the results in choosing the best option.

ES-401 Written Examination Question Worksheet Form ES-401-5 NUREG-1021 Revision-9 Examination Outline Cross-reference: Level RO SRO Tier # 3 Group # 3 K/A # G2.3.4 Importance Rating 2.5 Proposed Question 73:

An operator is scheduled to complete a valve lineup in an area where the radiation level is 50 mrem/hour and loose surface contamination is 500-750 dpm/cm2 (50-75 net counts per minute beta/gamma using a pancake frisker probe).

If the operators current Total Effective Dose Equivalent (TEDE) is 1400 mrem, how long can he work in this area and not exceed STPs Administrative Action Level (AAL) and what protective clothing requirements are required to be identified on the RWP?

A. 12 hours1.388889e-4 days <br />0.00333 hours <br />1.984127e-5 weeks <br />4.566e-6 months <br />; hood, coveralls, cotton liners, booties, gloves, rubber booties B. 2 hours2.314815e-5 days <br />5.555556e-4 hours <br />3.306878e-6 weeks <br />7.61e-7 months <br />; hood, coveralls, cotton liners, booties, gloves, rubber booties C. 12 hours1.388889e-4 days <br />0.00333 hours <br />1.984127e-5 weeks <br />4.566e-6 months <br />; no protective clothing is required D. 2 hours2.314815e-5 days <br />5.555556e-4 hours <br />3.306878e-6 weeks <br />7.61e-7 months <br />; no protective clothing is required Proposed Answer: D Explanation (Optional):

A - INCORRECT. 12 hours1.388889e-4 days <br />0.00333 hours <br />1.984127e-5 weeks <br />4.566e-6 months <br /> is determined using old AAL of 2000 mrem/yr and area is not a contamination area as defined by 0PGP03-ZR-0044 (100 net counts per minute).

B - INCORRECT. Stay time is correct but area is not a contamination area C - INCORRECT. Stay time is incorrect D - CORRECT. The current STP AAL is 1500 mrem/year. 1500-1400 = 100 mrem remaining. 100/50 mrem/hr = 2 hours2.314815e-5 days <br />5.555556e-4 hours <br />3.306878e-6 weeks <br />7.61e-7 months <br /> remaining. No protective clothing is required since area is not a contamination area.

0PGP03-ZR-0044, Contamination Control Program (Rev 14, Section 3.1);

Technical

Reference:

0PGP03-ZR-0050, Radiation Protection Program (Rev 7, Section 5.5.2)

Proposed references to be provided to applicants during examination:

None Learning Objective: (As available)

Question Source: Bank #

Modified Bank # STP - 581 (Note changes or attach parent)

New Question History: Last NRC Exam Cognitive Level: Memory or Fundamental Knowledge X Comprehension or Analysis

10 CFR Part 55 Content: 55.41 12 55.43 Comments:

KA: G2.3.4 - Knowledge of radiation exposure limits and contamination control, including permissible levels in excess of those authorized.

ES-401 Written Examination Question Worksheet Form ES-401-5 NUREG-1021 Revision-9 Examination Outline Cross-reference: Level RO SRO Tier # 3 Group # 4 K/A # G2.4.20 Importance Rating 3.3 Proposed Question 74:

In accordance with 0POP01-ZA-0018, Emergency Operating Procedure (EOP) Users Guide, which ONE of the following statements describes the proper use of CAUTIONS and NOTES?

CAUTIONS and NOTES within an EOP:

A. that are applicable to the ENTIRE procedure will appear on the Conditional Information Page (CIP).

B. always PRECEDE the step OR steps to which they apply.

C. are to be reviewed by the Unit Supervisor BEFORE beginning with step 1 of the procedure.

D. ONLY apply to the "Action/Expected Response" column items within a step.

Proposed Answer: B Explanation (Optional):

Section 4.3 states "Observe all CAUTIONS and NOTES which precede EOP step:

  • Cautions and notes always precede the step OR steps to which they apply.
  • Cautions and notes that precede the first step in an EOP may apply to the entire procedure.
  • Cautions and notes should be read aloud to the control room operators."

0POP01-ZA-0018, Emergency Operating Procedure Users Guide Technical

Reference:

(Rev 17, page 8 of 40, Section 4.3)

Proposed references to be provided to applicants during examination:

None Learning Objective: (As available)

Question Source: Bank # STP - 801 Modified Bank # (Note changes or attach parent)

New Question History: Last NRC Exam Cognitive Level: Memory or Fundamental Knowledge X Comprehension or Analysis 10 CFR Part 55 Content: 55.41 10 55.43 Comments:

KA: G2.4.20 - Knowledge of operational implications of EOP warnings, cautions, and notes.

ES-401 Written Examination Question Worksheet Form ES-401-5 NUREG-1021 Revision-9 Examination Outline Cross-reference: Level RO SRO Tier # 3 Group # 4 K/A # G2.4.32 Importance Rating 3.3 Proposed Question 75:

The following conditions exist in Unit 1:

  • Power is at 28% and being reduced to take the Unit offline for a short maintenance outage.
  • The oncoming shift, when testing the annunciator panels, notes that a large number of annunciator panels and their associated ICS Lampbox Mimic will NOT illuminate.

Which of the following actions should be taken in accordance with 0POP04-AN-0001, Loss of Control Room Annunciator Alarms?

A. Trip the reactor and enter 0POP05-EO-EO00, Reactor Trip or Safety Injection.

B. Continue the power reduction, but reduce the rate to less than 5% per hour.

C. Stabilize the plant at the current power level.

D. Prepare to place the Unit in Mode 3 within the next 6 hours6.944444e-5 days <br />0.00167 hours <br />9.920635e-6 weeks <br />2.283e-6 months <br />.

Proposed Answer: C Explanation (Optional):

Step 8 states MAINTAIN Current Plant Conditions Until Control Room Annunciator Alarms Are Restored.

0POP04-AN-0001, Loss Of Control Room Annunciator Alarms Technical

Reference:

(Rev. 13, Step 8)

Proposed references to be provided to applicants during examination:

None Learning Objective: (As available)

Question Source: Bank # STP - 357 Modified Bank # (Note changes or attach parent)

New Question History: Last NRC Exam Cognitive Level: Memory or Fundamental Knowledge X Comprehension or Analysis 10 CFR Part 55 Content: 55.41 10 55.43 Comments:

KA: G2.4.32 - Knowledge of operator response to loss of all annunciators.

ES-401, Rev. 9 STP SRO Written Examination Form ES-401-5 Question Worksheet Examination Outline Cross-

Reference:

Level SRO Tier # 1 Group # 1 K/A # 008.AA2.28 Importance Rating 3.9 Proposed Question 76:

While Unit 2 was operating at 100% power, a reactor trip and safety injection occurred.

The operators observe the following plant conditions when they reach the steps in 0POP05-EO-EO00, Reactor Trip or Safety Injection, that diagnose for procedure transition criteria:

  • RCS WR Pressure 1800 psig, slowly decreasing
  • RCS NR Thot 560°F, slowly decreasing
  • SG NR Levels 16%, slowly increasing
  • SG Pressures 1100 psig, stable
  • Main Steamline Radiation Level Normal
  • PRT Pressure 3 psig, increasing
  • PZR Level 28%, increasing
  • RCP Seal Injection Flow Normal
  • RCB Temperature 140°F, slowly increasing
  • RCB Pressure 0.5 psig, slowly increasing
  • RCB Humidity Increasing Which ONE of the following actions will the operators perform?

A. Transition to 0POP05-EO-EO20, Faulted Steam Generator Isolation B. Transition to 0POP05-EO-EO10, Loss of Reactor or Secondary Coolant C. Transition to 0POP05-EO-ES11, SI Termination D. Transition to 0POP05-EO-EO30, Steam Generator Tube Rupture Proposed Answer: ___B___

Explanation (Optional):

Candidate should determine that a PZR Code Safety Valve is stuck open (and not a PZR PORV since PZR PORVs are checked CLOSED at Step 9 of EO00). Either way, a loss of primary coolant requires entry into EO10.

A: INCORRECT. All SG pressures are stable therefore there is no faulted SG.

B: CORRECT.

C: INCORRECT. SI would not be terminated since RCS pressure is not stable or rising.

D: INCORRECT. Main steamline radiation levels are normal.

QDPS provides the following indications: RCS WR pressure, RCS NR Thot, SG NR levels, SG pressures, PZR level, RCB pressure.

Technical Reference(s): 0POP05-EO-EO00, Reactor Trip or Safety Injection Proposed references to be provided to applicants during examination: None Learning Objective: _________________________ (As available)

Question Source: Bank # _______

Modified Bank # ______ (Note changes or attach parent)

New ___X__

Question History: Last NRC Exam N/A Question Cognitive Level: Memory or Fundamental Knowledge _____

Comprehension or Analysis __X__

10 CFR Part 55 Content: 55.41 _____

55.43 _5___

Comments:

KA: 008 Pressurizer Vapor Space Accident, AA2.28 - Ability to determine and interpret the following as they apply to the Pressurizer Vapor Space Accident: Safety parameter display system indications.

ES-401, Rev. 9 STP SRO Written Examination Form ES-401-5 Question Worksheet Examination Outline Cross-

Reference:

Level SRO Tier # 1 Group # 1 K/A # 022.G2.4.30 Importance Rating 3.6 Proposed Question 77:

According to 10CFR50.72, Immediate Notification Requirements for Operating Nuclear Power Reactors, which ONE of the following situations would require the NRC be notified within one hour?

A. An SRO invokes 10CFR50.54(x) during a plant transient.

B. A valid ECCS actuation results in water being injected into the RCS.

C. A valid actuation of the Solid State Protection System (SSPS) results in a reactor trip from 100% power.

D. A plant shutdown is commenced under Tech Spec 3.0.3.

Proposed Answer: __A__

Explanation (Optional):

According to the referenced CFR, an immediate notification (within one hour) is required if a deviation from the Technical Specifications is made using 10CFR50.54(x). Answer B and C require the NRC be notified within 8 hours9.259259e-5 days <br />0.00222 hours <br />1.322751e-5 weeks <br />3.044e-6 months <br />, and D requires notification within 4 hours4.62963e-5 days <br />0.00111 hours <br />6.613757e-6 weeks <br />1.522e-6 months <br />.

Technical Reference(s): 10CFR50.72(b);

Conduct of Operations Manual, Chapter 2, Emergency Operations Outside of Design Basis Proposed references to be provided to applicants during examination: None Learning Objective: _________________________ (As available)

Question Source: Bank # _______

Modified Bank # ______ (Note changes or attach parent)

New ___X__

Question History: Last NRC Exam N/A Question Cognitive Level: Memory or Fundamental Knowledge __X__

Comprehension or Analysis _____

10 CFR Part 55 Content: 55.41 ____

55.43 _1__

Comments:

KA: 022 Loss of Reactor Coolant Makeup, G2.4.30 - Knowledge of which events related to system operations/status should be reported to outside agencies.

FYI - STP Ops Training Supervisor considers it NOT reasonable to expect an individual to have memorized the CFR to this degree. In practice, the SRO would use the station Reporting Manual to determine reporting requirements. Thus, STP recommends the candidate be provided the applicable sections of the Reporting Manual during the exam. STP also considers this a Comp/Analysis level question.

ES-401, Rev. 9 STP SRO Written Examination Form ES-401-5 Question Worksheet Examination Outline Cross-

Reference:

Level SRO Tier # 1 Group # 1 K/A # 026.AA2.01 Importance Rating 3.5 Proposed Question 78:

Unit 1 is operating at 100% power when the CCW SURGE TK LVL LO annunciator alarms. A Control Room operator notes the surge tank level steadily decreasing.

Actions by dispatched in-plant operators do not identify the source of the leak or restore surge tank level prior to CCW Surge Tank Low Level Non-Vital Supply Valves Isolation.

Subsequent to this isolation, Control Room operators report surge tank level is at 63%

and continues to decrease.

Which ONE of the following identifies the correct operator actions?

A. Open Normal Demineralized Water Makeup valve (LV-4501), ensure RCDT pumps in AUTO B. Secure Letdown, reduce Seal Injection to < 6 gpm, direct performance of local leak identification C. Verify CCW Surge Tank Low Level Non-Vital Supply Valves Isolation, secure Letdown, monitor Pressurizer level D. Trip the Reactor, stop RCPs, go to 0POP05-EO-EO00 Proposed Answer: __C___

Explanation (Optional):

A: Incorrect: LV-4501 should already be in the open position (Step 1 directs operators to check the valve open), and Step 7 directs operators to ensure RCDT pumps are Pull-to-Lock.

B: Incorrect: Letdown is secured in Step 3, local leak identification is directed in Step 8, but Seal Injection flow should be maintained 6-13 gpm as directed by Step 3.f.

C: Correct: Non-Vital Supply Valves are checked closed (or manually closed in RNO) in Step 2.a, Letdown is secured in Step 3, and PZR level is monitored in Step 6.

D: Incorrect: These actions are not required until Surge Tank Low Level Common Header Isolation signal is received at 61.5%

Technical Reference(s): 0POP04-CC-0001, Loss of Component Cooling Water (Rev 13)

Proposed references to be provided to applicants during examination: None Learning Objective: _________________________ (As available)

Question Source: Bank # _______

Modified Bank # _______ (Note changes or attach parent)

New ___X___

Question History: Last NRC Exam N/A Question Cognitive Level: Memory or Fundamental Knowledge _____

Comprehension or Analysis __X__

10 CFR Part 55 Content: 55.41 __5__

55.43 _____

Comments:

KA: 026 Loss of Component Cooling Water, AA2.01 - Ability to determine and interpret the following as they apply to the Loss of Component Cooling Water: Location of a leak in the CCWS

ES-401, Rev. 9 STP SRO Written Examination Form ES-401-5 Question Worksheet Examination Outline Cross-

Reference:

Level SRO Tier # 1 Group # 1 K/A # 057.AA2.19 Importance Rating 4.3 Proposed Question 79:

Unit 1 is conducting a plant startup. At 25% power, a loss of 120 VAC Vital Distribution Panel DP-1201 occurs.

The automatic response of the plant to this failure would require the crew to immediately:

A. place Deaerator Level Control Valve, LV-7406, in manual and maintain Deaerator level B. perform 0POP04-RP-0002, Loss of Automatic Pressurizer Level Control C. enter 0POP05-EO-EO00, Reactor Trip or Safety Injection D. control Steam Generator water levels manually.

Proposed Answer: __D__

Explanation (Optional):

A. Incorrect - This action is identified in Addendum 4 (Loss of DP1201) and is performed as a subsequent action.

B. Incorrect - Step 9 of Tech Ref states to perform off-normal procedures for applicable failures as manpower permits.

C. Incorrect - The loss of one Class 1E 120 VAC distribution panel will not directly trip the unit but prompt action is required to preclude key control parameters from exceeding their reactor trip setpoints.

D. Correct - This immediate action is required regardless of what 120 VAC distribution panel failed.

Technical Reference(s): 0POP04-VA-0001, Loss Of 120 VAC Class Vital Distribution (Rev 18)

Proposed references to be provided to applicants during examination: None

Learning Objective: _________________________ (As available)

Question Source: Bank # _______

Modified Bank # _______ (Note changes or attach parent)

New ___X___

Question History: Last NRC Exam N/A Question Cognitive Level: Memory or Fundamental Knowledge _____

Comprehension or Analysis __X__

10 CFR Part 55 Content: 55.41 __5__

55.43 _____

Comments:

Original KA (057.AA2.09 - Ability to determine and interpret the following as they apply to the Loss of Vital AC Instrument Bus: Tave and Tref chart recorder) was REJECTED because this is not an SRO level topic at STP.

KA: 057 Loss of Vital AC Instrument Bus, AA2.19 - Ability to determine and interpret the following as they apply to the Loss of Vital AC Instrument Bus: The plant automatic actions that will occur on the loss of a vital ac instrument bus.

ES-401, Rev. 9 STP SRO Written Examination Form ES-401-5 Question Worksheet Examination Outline Cross-

Reference:

Level SRO Tier # 1 Group # 1 K/A # W/E04.G2.3.10 Importance Rating 3.3 Proposed Question 80:

Unit 2 has experienced a large break LOCA outside containment. Station operators have completed the required actions of 0POP05-EO-EO00, Reactor Trip or Safety Injection, and have transitioned to 0POP05-EO-EC12, LOCA Outside Containment. If the source of the leak cannot be identified and/or isolated by the completion of 0POP05-EO-EC12, the SRO should direct the crew to:

A. Go to procedure 0POP05-E0-EC11, Loss of Emergency Coolant Recirculation, Step 1, in order to delay depletion of the RWST by adding makeup and reducing outflow.

B. Restart procedure 0POP05-E0-EC12 at Step 1 and re-attempt to isolate the leak while monitoring Critical Safety Functions.

C. Go to procedure 0POP05-E0-ES00, Rediagnosis, Step 1, to determine the most appropriate post-accident recovery procedure.

D. Return to procedure 0POP05-E0-EO10, Loss of Reactor or Secondary Coolant, Step 1, to recover from a loss of reactor coolant.

Proposed Answer: __A__

Explanation (Optional):

A: Correct - as identified in 0POP05-EO-EC12, Step 6 (RNO).

B: Incorrect - While monitoring of CSF is correct, EC12 directs going to EC11.

C: Incorrect - While ES00 is used to determine the most appropriate post-accident recovery procedure, EC12 directs going to EC11.

D: Incorrect - EO10 is used to recover from a loss of reactor coolant inside containment Technical Reference(s): 0POP05-EO-EO00 (Rev 17) 0POP05-EO-EC12 (Rev 7) 0POP05-E0-EC11 (Rev 13)

Proposed references to be provided to applicants during examination: None Learning Objective: _________________________ (As available)

Question Source: Bank # _______

Modified Bank # _______ (Note changes or attach parent)

New ___X___

Question History: Last NRC Exam N/A Question Cognitive Level: Memory or Fundamental Knowledge __X__

Comprehension or Analysis _____

10 CFR Part 55 Content: 55.41 _____

55.43 __X__

Comments:

Original KA (W/E04 - LOCA Outside Containment, G2.3.9 - Knowledge of the process for performing a containment purge) was REJECTED because it is not applicable during a LOCA outside of containment.

KA: W/E04 - LOCA Outside Containment; G2.3.10 - Ability to perform procedures to reduce excessive levels of radiation and guard against personnel exposure.

ES-401, Rev. 9 STP SRO Written Examination Form ES-401-5 Question Worksheet Examination Outline Cross-

Reference:

Level SRO Tier # 1 Group # 1 K/A # W/E05.G2.4.8 Importance Rating 3.7 Proposed Question 81:

A scenario has developed requiring the SRO to enter the EOPs. During the implementation of the EOPs, abnormal indications develop involving the Reactor Coolant Pumps (RCPs) and the potential for a loss of forced core cooling flow.

In accordance with 0POP01-ZA-0018, EOP Users Guide, in this situation, the SRO:

A. Must complete the EOPs actions and then implement the RCP Off-Normal Procedure.

B. Must concurrently implement the EOPs and the RCP Off-Normal Procedure.

C. Should concurrently implement the RCP Off-Normal Procedure if resources permit and it does not conflict with the EOPs.

D. Should concurrently implement the RCP Off-Normal Procedure and perform the EOP actions if resources permit and they do not conflict with the RCP Off-Normal.

Proposed Answer: __C__

Explanation (Optional):

Per procedure 0POP01ZA0018, 4.25.4 states actions should be taken per Off Normal Operating Procedures and Annunciator Response Procedures that DO NOT conflict with the actions of the EOPs if adequate resources are available. The Off Normal Operating Procedure or Annunciator Response Procedure should be entered and procedure steps followed. (e.g., IF during the performance of the EOPs there are indications of abnormal RCP conditions, THEN the RCP Off Normal Operating Procedure SHOULD be entered.)

Technical Reference(s): 0POP01ZA0018, EOP Users Guide (Rev 17, Section 4.25.4)

Proposed references to be provided to applicants during examination: None Learning Objective: _________________________ (As available)

Question Source: Bank # _______

Modified Bank # ______ (Note changes or attach parent)

New ___X__

Question History: Last NRC Exam N/A Question Cognitive Level: Memory or Fundamental Knowledge __X__

Comprehension or Analysis _____

10 CFR Part 55 Content: 55.41 _____

55.43 _5___

Comments:

KA: W/E05 Inadequate Heat Transfer - Loss of Secondary Heat Sink, G2.4.8 - Knowledge of how the event-based emergency/abnormal operating procedures are used in conjunction with the symptom-based EOPs.

ES-401, Rev. 9 STP SRO Written Examination Form ES-401-5 Question Worksheet Examination Outline Cross-

Reference:

Level SRO Tier # 1 Group # 2 K/A # 005.AA2.03 Importance Rating 4.4 Proposed Question 82:

Given the following:

  • 0100: A Shutdown Bank control rod was discovered to be stuck in the fully withdrawn position due to some type of mechanical interference. The rod is declared inoperable.
  • 0200: A second Shutdown Bank rod dropped into the core.
  • 0230: Efforts to recover the dropped rod are unsuccessful. The rod is declared inoperable.

Based on these conditions, which ONE of the below actions is required by Tech Specs?

A. Must be in Mode 3 no later than 0700.

B. Must be in Mode 3 no later than 0800.

C. May remain in Mode 1, however power must be at less than 75% rated thermal power no later than 0300.

D. May remain in Mode 1, however power must be at less than 75% rated thermal power no later than 0330.

Proposed Answer: __A__

Explanation (Optional):

In this situation, TS 3.1.3, Movable Control Assemblies - Group Height, applies. With the two rods inoperable LCO action statements 3.1.3.1.a and 3.1.3.1.b must be entered. LCO action statements 3.1.3.1.c and 3.1.3.1.d do not apply. Of the two action statements, 3.1.3.1.a is the most restrictive and it requires the plant be in hot standby (Mode 3) within 6 hours6.944444e-5 days <br />0.00167 hours <br />9.920635e-6 weeks <br />2.283e-6 months <br />. Since the rod was declared inoperable at 0100, the plant must be in Mode 3 no later than 0700 making A the correct answer.

Technical Reference(s): Technical Specification 3/4.1.3 Proposed references to be provided to applicants during examination: Applicable TS to be provided as part of separate reference package Learning Objective: _________________________ (As available)

Question Source: Bank # _______

Modified Bank # _______ (Note changes or attach parent)

New ___X___

Question History: Last NRC Exam N/A Question Cognitive Level: Memory or Fundamental Knowledge _____

Comprehension or Analysis __X__

10 CFR Part 55 Content: 55.41 _____

55.43 _2___

Comments:

KA: 005 Inoperable/Stuck Control Rod, AA2.03 - Determine and interpret required actions if more than one rod is stuck or inoperable.

ES-401, Rev. 9 STP SRO Written Examination Form ES-401-5 Question Worksheet Examination Outline Cross-

Reference:

Level SRO Tier # 1 Group # 2 K/A # 061.AA2.03 Importance Rating 3.3 Proposed Question 83:

During an accident condition, RCB High Range Area Rad Monitors RT-8050 and RT-8051 begin trending up. Which ONE of the below correctly describes the response of these monitors?

A. If either one reaches its ALERT setpoint, it will alarm and initiate a Containment Ventilation Isolation (CVI).

B. If either one reaches its HIGH setpoint, it will alarm and initiate a Containment Ventilation Isolation (CVI).

C. If either one reaches its ALERT setpoint, it will alarm only.

D. Neither one will alarm until the HIGH setpoint is reached.

Proposed Answer: __C__

Explanation (Optional):

Per the Tech Ref, no actuations are associated with these monitors so answers A and B are incorrect. Answer D is incorrect because either monitor will alarm at the ALERT setpoint. C is the only correct answer.

Technical Reference(s): LOT202.41.HO02, Process & Effluent Radiation Monitoring, Operability Determination Guides Student Handout (Rev 13, page 10)

Proposed references to be provided to applicants during examination: None Learning Objective: _________________________ (As available)

Question Source: Bank # _______

Modified Bank # _______ (Note changes or attach parent)

New ___X___

Question History: Last NRC Exam N/A Question Cognitive Level: Memory or Fundamental Knowledge __X__

Comprehension or Analysis _____

10 CFR Part 55 Content: 55.41 ______

55.43 __12__

Comments:

KA: 061 Area Radiation Monitoring (ARM) System Alarms, AA2.03 - Ability to determine and interpret the following as they apply to the Area Radiation Monitoring (ARM) System Alarms:

Setpoints for alert and high alarms

ES-401, Rev. 9 STP SRO Written Examination Form ES-401-5 Question Worksheet Examination Outline Cross-

Reference:

Level SRO Tier # 1 Group # 2 K/A # 068.AA2.08 Importance Rating 4.1 Proposed Question 84:

The Control Room is being evacuated due to a fire in the Relay Room.

Which ONE of the following actions is completed to ensure an uncontrolled RCS cooldown does not occur, and from what location is it completed?

A. Isolate Main Steam System; Auxiliary Shutdown Panel B. Isolate Main Steam System; Control Room C. Close Pressurizer PORV Block Valves; Auxiliary Shutdown Panel D. Close Pressurizer PORV Block Valves; Control Room Proposed Answer: __B___

Explanation (Optional):

Step 2 of the Tech Ref is to isolate the main steam system prior to exiting the CR. The basis for this action is to ensure that an uncontrolled cooldown does not occur and it can be accomplished within the required time frame, thus Answer B is the only correct answer. Closing PZR PORV block valves is done to reduce the potential for uncontrolled RCS depressurization and loss of inventory (PCV0655A and PCV0656A may spuriously open due to circuitry affected in the Control Room).

Technical Reference(s): 0POP04-ZO-0001, Control Room Evacuation (Rev. 25, Page 130 of 205) and Addendum 20 Basis (Basis Page 4 of 78)

Proposed references to be provided to applicants during examination: None Learning Objective: _________________________ (As available)

Question Source: Bank # _______

Modified Bank # _______ (Note changes or attach parent)

New ___X___

Question History: Last NRC Exam N/A Question Cognitive Level: Memory or Fundamental Knowledge _____

Comprehension or Analysis __X__

10 CFR Part 55 Content: 55.41 _____

55.43 __X__

Comments:

KA: 068 Control Room Evacuation, AA2.08 - Ability to determine and interpret the following as they apply to the Control Room Evacuation: S/G pressure Candidate must recognize that any escape of steam from the SGs reduces SG pressure, thus lowering Tsat. Lowering secondary temperature will increase the heat transfer rate from the primary to the secondary, thus causing a cooldown of the primary.

ES-401, Rev. 9 STP SRO Written Examination Form ES-401-5 Question Worksheet Examination Outline Cross-

Reference:

Level SRO Tier # 1 Group # 2 K/A # W/E03 G2.4.21 Importance Rating 4.3 Proposed Question 85:

Unit 2 has experienced a small break LOCA. Operators have just transitioned from 0POP05-EO-EO10, Loss of Reactor or Secondary Coolant, to 0POP05-EO-ES12, Post LOCA Cooldown and Depressurization, Step 1. The STA reports the following conditions as read from QDPS:

  • Core Exit T/C reading is 600 ºF
  • Subcooling is indicating 0 ºF
  • RVWL is indicating 20% Plenum Level Which ONE of the following identifies the correct Core Cooling CSF Status Tree color and what procedure will the Unit Supervisor be required to implement?

A. ORANGE; Go To 0POP05-EO-FRC2, Response to Degraded Core Cooling.

B. ORANGE, Go To 0POP05-EO-FRC3, Response to Saturated Core Cooling.

C. YELLOW, Continue with 0POP05-EO-ES12.

D. YELLOW, Go To 0POP05-EO- FRC3, Response to Saturated Core Cooling Proposed Answer: __C___

Explanation (Optional):

Candidate must analyze current conditions and compare them to decision points in the Core Cooling CSF Status Tree. Although the question stem doesnt indicate whether containment is adverse or not, it is not required since subcooling margin is below either number (35°F or 45°F for adverse conditions).

A: Incorrect. The status tree exits to a YELLOW condition with RVWL plenum greater than or equal to 20%

B: Incorrect. Same as above, plus FRC3 is only implemented from a YELLOW

C: Correct. Core Cooling CSF Status Tree decision points are 1) core exit T/Cs < 1200°F -

yes. Then 2) RCS subcooling based on core exit T/Cs > 35°F - no. Then 3) RVWL plenum >

or equal to 20% - yes. Then go to EO-FRC3 (YELLOW). However, the procedure in effect is not exited for a YELLOW status tree indicator.

D: Incorrect. Color is correct, but procedure in effect is not exited for a YELLOW indicator.

Technical Reference(s): 0POP05-EO-EO10, Loss of Reactor or Secondary Coolant (R17) 0POP05-EO-ES12, Post LOCA Cooldown and Depressurization (R11) 0POP05-EO-FO02, Core Cooling CSF Status Tree (R1)

Proposed references to be provided to applicants during examination: None Learning Objective: _________________________ (As available)

Question Source: Bank # _______

Modified Bank # _______ (Note changes or attach parent)

New ___X___

Question History: Last NRC Exam N/A Question Cognitive Level: Memory or Fundamental Knowledge _____

Comprehension or Analysis __X__

10 CFR Part 55 Content: 55.41 _____

55.43 __X__

Comments:

KA: W/E03 (LOCA Cooldown and Depressurization) G2.4.21 - Knowledge of the parameters and logic used to assess the status of safety functions including:

1. Reactivity control
2. Core cooling and heat removal
3. Reactor coolant system integrity
4. Containment conditions
5. Radioactivity release control

ES-401, Rev. 9 STP SRO Written Examination Form ES-401-5 Question Worksheet Examination Outline Cross-

Reference:

Level SRO Tier # 2 Group # 1 K/A # 012.A2.02 Importance Rating 3.9 Proposed Question 86:

Given the following:

  • Unit 2 is operating at 30% steady state reactor power
  • An I&C technician receives permission to perform a calibration on Power Range Channel N41
  • The I&C technician mistakenly pulls the control power fuses on Power Range Channel N42; then, realizing his mistake, he re-inserts the fuses for N42 and pulls the control power fuses for the correct channel, N41.

Which procedure should the Unit Supervisor implement AND what is the correct reason?

A. 0POP05-EO-EO00, Reactor Trip or Safety Injection, because an automatic reactor trip has occurred on Power Range High Flux, High Setpoint Trip B. 0POP05-EO-EO00, Reactor Trip or Safety Injection, because an automatic reactor trip has occurred on Power Range Positive Rate.

C. 0POP04-NI-0001, Nuclear Instrument Malfunction, to ensure PR N42 is in service and operable.

D. 0POP04-NI-0001, Nuclear Instrument Malfunction, to realign SSPS input logic to account for PR N41 being removed from service.

Proposed Answer: ___B___

Explanation (Optional):

0POP04-NI-0001, Nuclear Instrument Malfunction, Addendum 3, PR NI Malfunction (Step 15.g) has the control power fuses pulled, then checks the bistables tripped per Addendum 6.

Addendum 6 lists 4 bistables that will trip: PR LO (reduced high flux trip setpoint when P < P-10), PR HI (normal High Flux trip thats enabled at P > P-10), Positive Rate, and OTDT Loop 1.

When the control power fuses for N42 are reinstalled, clears all the trip signals except for the positive rate trip. Theres a reset switch for this trip on the NI cabinets in the control room.

Thus, when control power fuses for N41 are pulled, 2 of 4 trip signals are present for Positive Rate.

Technical Reference(s): LOT201.16.HO.01, Excore Nuclear Instrumentation Student Handout (Rev. 12, page 22) 0POP04-NI-0001, Nuclear Instrument Malfunction (Rev 11)

Proposed references to be provided to applicants during examination: None Learning Objective: _________________________ (As available)

Question Source: Bank # STP - 905 Modified Bank # _______ (Note changes or attach parent)

New _______

Question History: Last NRC Exam N/A Question Cognitive Level: Memory or Fundamental Knowledge ______

Comprehension or Analysis __X___

10 CFR Part 55 Content: 55.41 _____

55.43 __5__

Comments:

KA: 012 Reactor Protection System, A2.02 - Ability to (a) predict the impacts of the following malfunctions or operations on the RPS; and (b) based on those predictions, use procedures to correct, control, or mitigate the consequences of those malfunctions or operations: Loss of instrument power.

ES-401, Rev. 9 STP SRO Written Examination Form ES-401-5 Question Worksheet Examination Outline Cross-

Reference:

Level SRO Tier # 2 Group # 1 K/A # 026.G2.1.7 Importance Rating 4.4 Proposed Question 87:

You are the SRO on shift when the following occur IN SEQUENCE:

  • A Loss of Coolant Accident (LOCA) occurs
  • All ESF equipment is functioning as designed
  • Containment Phase A Isolation is reset
  • ESF Load Sequencers are reset in the Control Room only
  • Containment Pressure increases to 9.8 psig.
  • A Reactor Operator reports that no Containment Spray Pumps are running, but their discharge valves are open.

Based on these conditions, you inform the Reactor Operator that the Containment Spray Pumps ____________:

A. should not be running yet.

B. should be running and to start them by placing BOTH Containment Spray Manual Actuation Switches to ACTUATE.

C. should be running and to start them by placing EITHER Containment Spray Manual Actuation Switch to ACTUATE.

D. should be running and to start them manually using their respective pump control switch.

Proposed Answer: __D___

Explanation (Optional):

The student must recognize that the ESF Load Sequencers were reset before Containment pressure increased to above the Containment Spray actuation setpoint thereby disabling the actuation logic both automatically and manually. Thus the Containment Spray Pumps would have to be stared manually with their control switches.

Technical Reference(s): 0POP05-EO-FRZ1, Response to High Containment Pressure (Rev 6, Step 3)

Proposed references to be provided to applicants during examination: None Learning Objective: _________________________ (As available)

Question Source: Bank # _______

Modified Bank # _______ (Note changes or attach parent)

New ___X___

Question History: Last NRC Exam N/A Question Cognitive Level: Memory or Fundamental Knowledge _____

Comprehension or Analysis __X__

10 CFR Part 55 Content: 55.41 _____

55.43 _5___

Comments:

KA: 026 Containment Spray, G2.1.7 - Ability to evaluate plant performance and make operational judgments based on operating characteristics, reactor behavior, and instrument interpretation.

ES-401, Rev. 9 STP SRO Written Examination Form ES-401-5 Question Worksheet Examination Outline Cross-

Reference:

Level SRO Tier # 2 Group # 1 K/A # 059.A2.04 Importance Rating 3.4 Proposed Question 88:

Unit 1 has experienced an accident. The crew is currently implementing 0POP05-EO-FRH1, Response to Loss of Secondary Heat Sink.

Current plant conditions are:

  • RCS pressure = 2280 psig, increasing
  • RCS temperature = 570 °F, increasing
  • RCB pressure = 0.4 psig, stable
  • SG A WR level = 52%, decreasing
  • SG B WR level = 49%, decreasing
  • SG C WR level = 51 %, decreasing
  • SG D WR level = 55%, decreasing
  • NO AFW flow can be established Based on the current plant conditions and in accordance with 0POP05-EO-FRH1, the Unit Supervisor should establish a heat sink by:

A. Attempting to feed SGs with the Main Feedwater System B. Attempting to feed SGs with the Condensate System C. Initiating RCS Bleed and Feed.

D. Depressurizing all SGs.

Proposed Answer: ___A___

Explanation (Optional):

With the given plant conditions, candidate must first be able to evaluate if those conditions warrant bleed and feed (SG WR levels < 50% in at least two SGs OR PZR pressure > 2335 psig). Once they determine conditions dont warrant bleed and feed, they then must know what

method is to be used for a heat sink under these conditions. Distracters B and D are both viable options that are used under other circumstances.

Technical Reference(s): 0POP05-EO-FRH1, Response to Loss of Secondary Heat Sink (Rev 14)

Proposed references to be provided to applicants during examination: None Learning Objective: _________________________ (As available)

Question Source: Bank # _______

Modified Bank # _______ (Note changes or attach parent)

New _______

Question History: Last NRC Exam N/A Question Cognitive Level: Memory or Fundamental Knowledge ______

Comprehension or Analysis __X___

10 CFR Part 55 Content: 55.41 _____

55.43 __5__

Comments:

KA: 059 Main Feedwater (MFW) System, A2.04 - Ability to (a) predict the impacts of the following malfunctions or operations on the MFW; and (b) based on those predictions, use procedures to correct, control, or mitigate the consequences of those malfunctions or operations: Feeding a dry SG.

ES-401, Rev. 9 STP SRO Written Examination Form ES-401-5 Question Worksheet Examination Outline Cross-

Reference:

Level SRO Tier # 2 Group # 1 K/A # 062.A2.01 Importance Rating 3.9 Proposed Question 89:

A loss of all AC power has occurred.

  • The crew has completed the immediate actions of 0POP05-EO-EC00, Loss of All AC Power.
  • Buses 1G5 and 12K3 are energized
  • NO other busses are energized Based on these conditions, the SRO should direct the crew to:

A. Exit 0POP05-EO-EC00 because several buses have AC power and attempt to restore the 1G8 bus per 0POP02-DB-0003, Balance of Plant Diesel Generator.

B. Continue with 0POP05-EO-EC00 and attempt to restore the 1G8 bus per 0POP02-DB-0003, Balance of Plant Diesel Generator.

C. Exit 0POP05-EO-EC00 because several buses have AC power and attempt to restore the 1G8 bus per 0POP02-DB-0005, Technical Support Center Diesel Generator.

D. Continue with 0POP05-EO-EC00 and attempt to restore the 1G8 bus per 0POP02-DB-0005, Technical Support Center Diesel Generator.

Proposed Answer: __D__

Explanation (Optional):

The next step in procedure 0POP05-E0-EC00 is step 3, which is to establish RCP flow using the PDP pump. The PDP pump is powered from bus 1G8 via the TSC diesel generator, which did not start as implied in the stem. This makes D correct. A and C are incorrect because you cannot exit procedure 0POP05-E0-EC00 until several steps later (when vital power is restored, etc.). The PDP is on 1G8, not 1G5, and so B is also incorrect because the BOP generator powers 1G5.

Technical Reference(s): 0POP05-E0-EC00, Loss of All AC Power (Rev 16, page 5);

0POP02-DB-0005, Technical Support Center Diesel Generator (Rev 27)

Proposed references to be provided to applicants during examination: None Learning Objective: _________________________ (As available)

Question Source: Bank # _______

Modified Bank # _______ (Note changes or attach parent)

New ___X___

Question History: Last NRC Exam N/A Question Cognitive Level: Memory or Fundamental Knowledge _____

Comprehension or Analysis __X__

10 CFR Part 55 Content: 55.41 _____

55.43 __5__

Comments:

Original KA (062 AC Electrical Distribution, A2.14 - Performance of ground isolation procedures: determination of their effect on interface systems) was REJECTED because STP does not have ground isolation procedures KA: 062 AC Electrical Distribution, A2.01 - Ability to (a) predict the impacts of the following malfunctions or operations on the AC Distribution System; and (b) based on those predictions, use procedures to correct, control, or mitigate the consequences of those malfunctions or operations: type of loads that, if de-energized, would degrade or hinder plant operation.

ES-401, Rev. 9 STP SRO Written Examination Form ES-401-5 Question Worksheet Examination Outline Cross-

Reference:

Level SRO Tier # 2 Group # 1 K/A # 073.G2.3.11 Importance Rating 3.2 Proposed Question 90:

During Refueling operations, a fuel assembly is dropped in Containment. Containment Ventilation Isolation (CVI) has automatically actuated from HIGH Alarms on RT-8012 and RT-8013.

Which ONE of the below describes the operation of the Containment Carbon Filter Units?

A. The Carbon Filter Units are started automatically to filter particulate radiation and radioiodine from the Containment atmosphere.

B. The Carbon Filter Units are started automatically to filter gaseous and particulate radiation from the Containment atmosphere.

C. The Carbon Filter Units are started manually to filter particulate radiation and radioiodine from the Containment atmosphere.

D. The Carbon Filter Units are started manually to filter gaseous and particulate radiation from the Containment atmosphere.

Proposed Answer: __C__

Explanation (Optional):

UFSAR states the charcoal absorbers remove airborne radioiodine from the air stream.

0POP04-FH-0001 informs operators to start Carbon Filter Units at Step 8 and IAW 0POP02-HC-0001, Containment HVAC. Per HC-0001, the Carbon Filter Units are started manually.

These filter units are not Safety Related thus they are not part of the ESF actuation logic.

Technical Reference(s): UFSAR Chapter 9, Section 9.4.5.2.3; 0POP04-FH-0001, Fuel Handling Accident (Rev. 7, Step 8);

0POP02-HC-0001, Containment HVAC (Rev. 12)

Proposed references to be provided to applicants during examination: None Learning Objective: _________________________ (As available)

Question Source: Bank # _______

Modified Bank # _______ (Note changes or attach parent)

New ___X___

Question History: Last NRC Exam N/A Question Cognitive Level: Memory or Fundamental Knowledge __X__

Comprehension or Analysis _____

10 CFR Part 55 Content: 55.41 _____

55.43 _4, 7_

Comments:

KA: 073 Process Radiation Monitoring System, G2.3.11 - Ability to control radiation releases.

ES-401, Rev. 9 STP SRO Written Examination Form ES-401-5 Question Worksheet Examination Outline Cross-

Reference:

Level SRO Tier # 2 Group # 2 K/A # 011.A2.11 Importance Rating 3.6 Proposed Question 91:

Unit 2 is in Hot Standby with normal operating pressure when Pressurizer Level instrument LT-465 fails. The Technical Specifications require that:

A. no action be taken.

B. the channel be tripped within 72 hrs.

C. the channel be restored to operable status within 30 days or be in Hot Shutdown within the following 6 hrs.

D. the channel be restored to operable status within 48 hours5.555556e-4 days <br />0.0133 hours <br />7.936508e-5 weeks <br />1.8264e-5 months <br /> or be in Hot Shutdown within the following 12 hrs.

Proposed Answer: __A___

Explanation (Optional):

TS 3.3.1, Reactor Trip System Instrumentation, Table 3.3-1, contains PZR Water Level-High, but is only applicable in Mode 1. Stem of question states unit is in Mode 3, thus A is correct in that no action is required to be taken.

Technical Reference(s): TS 3.3.1 for RPS table Proposed references to be provided to applicants during examination: Applicable TS to be provided as part of separate reference package Learning Objective: _________________________ (As available)

Question Source: Bank # _______

Modified Bank # _______ (Note changes or attach parent)

New ___X___

Question History: Last NRC Exam N/A Question Cognitive Level: Memory or Fundamental Knowledge _____

Comprehension or Analysis __X__

10 CFR Part 55 Content: 55.41 ____

55.43 __5_

Comments:

Original KA (011 Pressurizer Level Control System, A2.08 - Loss of level compensation) was rejected because STP does not have level compensation for PZR level.

KA: 011 Pressurizer Level Control System, A2.11 - Ability to (a) predict the impacts of the following malfunctions or operations on the PZR Level Control System; and (b) based on those predictions, use procedures to correct, control, or mitigate the consequences of those malfunctions or operations: Failure of PZR Level Instrument-low.

ES-401, Rev. 9 STP SRO Written Examination Form ES-401-5 Question Worksheet Examination Outline Cross-

Reference:

Level SRO Tier # 2 Group # 2 K/A # 029.A2.04 Importance Rating 3.2 Proposed Question 92:

Given the following:

  • Plant is at 100% power steady state operation.
  • Preparations for performing a containment purge are in progress.
  • Noble gas concentration inside the RCB is 5.2E-04 uCi/cc.

Which ONE of the following identifies the procedure that should be used for the purge AND the actions that should be taken to prevent the actuation of an ESF Containment Ventilation Isolation (CVI) during the containment purge?

A. 0POP02-HC-0002, NORMAL CONTAINMENT PURGE; Increase the High alarm setpoint of RT 8012 & 8013 (RCB Purge Monitors).

B. 0POP02-HC-0003, SUPPLEMENTARY CONTAINMENT PURGE; Increase the High alarm setpoint of RT 8012 & 8013 (RCB Purge Monitors).

C. 0POP02-HC-0002, NORMAL CONTAINMENT PURGE; Increase the High alarm setpoint on RT-8011 (Containment atmosphere radiation monitor).

D. 0POP02-HC-0003, SUPPLEMENTARY CONTAINMENT PURGE; Increase the High alarm setpoint on RT-8011 (Containment atmosphere radiation monitor).

Proposed Answer: __B__

Explanation (Optional):

0POP02HC003, SUPPLEMENTARY CONTAINMENT PURGE is the procedure to be used during purge of containment at power, eliminating A and C choices. To prevent CVI isolation, the high alarm setpoint for 8012/8013 must be increased IAW procedure 0POP02-HC-0003, eliminating choice D.

Technical Reference(s): 0POP02-HC-0003, Supplementary Containment Purge (Rev 17) 0POP02-HC-0002, Normal Containment Purge (Rev 10)

Proposed references to be provided to applicants during examination: None Learning Objective: _________________________ (As available)

Question Source: Bank # _______

Modified Bank # _______ (Note changes or attach parent)

New ___X___

Question History: Last NRC Exam N/A Question Cognitive Level: Memory or Fundamental Knowledge _____

Comprehension or Analysis __X__

10 CFR Part 55 Content: 55.41 ____

55.43 _5__

Comments:

KA: 029 Containment Purge System, A2.04 - Ability to (a) predict the impacts of the following malfunctions or operations on the Containment Purge System; and (b) based on those predictions, use procedures to correct, control, or mitigate the consequences of those malfunctions or operations: Health physics sampling of containment atmosphere.

ES-401, Rev. 9 STP SRO Written Examination Form ES-401-5 Question Worksheet Examination Outline Cross-

Reference:

Level SRO Tier # 2 Group # 2 K/A # 033.G2.4.37 Importance Rating 3.5 Proposed Question 93:

A severe hurricane initiated a plant transient that has resulted in a complete loss of Spent Fuel Pool Cooling during a Unit 1 outage. Pool boiling over the last few hours has resulted in a loss of water inventory and fuel damage is now beginning to occur.

The Shift Supervisor has assumed the role of Emergency Director and does not anticipate being relieved for the next few hours due to the weather conditions. Based on the radiological release now in progress, the Emergency Director has declared a Site Area Emergency.

In accordance with 0ERP01-ZV-SH01, Shift Supervisor, the Emergency Director may delegate the authority to:

A. approve required notifications to the state and county.

B. approve exposures in excess of those contained in 10CFR20, Standards for Protection Against Radiation.

C. authorize the use of potassium iodide (KI) pills.

D. request federal assistance.

Proposed Answer: D Explanation (Optional):

According to the South Texas Project Emergency Plan, answers A, B, and C may not be delegated. Answer D may be delegated making it the only correct answer.

Technical Reference(s): 0OERP01-ZV-SH01, Shift Supervisor, steps 5.5 and 5.6.

Proposed references to be provided to applicants during examination: None

Learning Objective: _________________________ (As available)

Question Source: Bank # _______

Modified Bank # _______ (Note changes or attach parent)

New ___X___

Question History: Last NRC Exam N/A Question Cognitive Level: Memory or Fundamental Knowledge __X__

Comprehension or Analysis _____

10 CFR Part 55 Content: 55.41 ____

55.43 _5__

Comments:

KA: 033 Spent Fuel Pool Cooling System, G2.4.37 - Knowledge of the lines of authority during an emergency.

ES-401, Rev. 9 STP SRO Written Examination Form ES-401-5 Question Worksheet Examination Outline Cross-

Reference:

Level SRO Tier # 3 Group #

K/A # G2.1.5 Importance Rating 3.4 Proposed Question 94:

One of the shift staffing requirements documented on the Shift Supervisor Shift Turnover Checklist is a verification of meeting the minimum fire brigade staffing. The minimum fire brigade staffing is defined in:

A. the Technical Requirements Manual (TRM).

B. the Technical Specifications.

C. procedure 0POP01-ZQ-0022, Plant Operation Shift Routines.

D. the Conduct of Operations Manual.

Proposed Answer: __A__

Explanation (Optional):

The minimum staffing requirement is found in TRM 6.2.2.e, Fire Brigade. This makes A correct.

The actual staffing requirement is referenced in the other documents but the specific number is not provided.

Technical Reference(s): TRM 6.2.2.e, Fire Brigade Proposed references to be provided to applicants during examination: None Learning Objective: _________________________ (As available)

Question Source: Bank # _______

Modified Bank # _______ (Note changes or attach parent)

New ___X___

Question History: None Last NRC Exam N/A Question Cognitive Level: Memory or Fundamental Knowledge __X__

Comprehension or Analysis _____

10 CFR Part 55 Content: 55.41 ____

55.43 _1__

Comments:

KA: Conduct of Operations, G2.1.5 - Ability to locate and use procedures and directives related to shift staffing and activities.

ES-401, Rev. 9 STP SRO Written Examination Form ES-401-5 Question Worksheet Examination Outline Cross-

Reference:

Level SRO Tier # 3 Group #

K/A # G2.1.22 Importance Rating 3.3 Proposed Question 95:

Unit 1 is being started up shortly after a human performance error resulted in a reactor trip from full power following 401 continuous days on-line. Currently, Tave is 567 °F and total core heat generation is 5.5% (4.7% from thermal power generation and 0.8% from decay heat).

Based on these conditions, according to the technical specifications the reactor is now in Mode:

A. 1 B. 2 C. 3 D. 4 Proposed Answer: B Explanation (Optional):

The reactor is in Mode 2 because reactor power is less than 5% (decay heat is excluded from determining the Mode) and Tave is greater than 350 degrees. Answer A is incorrect because reactor thermal power is less than 5%. Answer C is incorrect because Keff is greater than 0.99. Answer D is incorrect because Tave is greater than 350 degrees.

Technical Reference(s): Technical Specification Table 1.2 Proposed references to be provided to applicants during examination: None Learning Objective: _________________________ (As available)

Question Source: Bank # _______

Modified Bank # _______ (Note changes or attach parent)

New ___X___

Question History: Last NRC Exam N/A Question Cognitive Level: Memory or Fundamental Knowledge _____

Comprehension or Analysis __X__

10 CFR Part 55 Content: 55.41 ____

55.43 _2__

Comments:

KA: Conduct of Operations, G2.1.22 - Ability to determine Mode of Operation.

ES-401, Rev. 9 STP SRO Written Examination Form ES-401-5 Question Worksheet Examination Outline Cross-

Reference:

Level SRO Tier # 3 Group #

K/A # 2.2.26 Importance Rating 3.7 Proposed Question 96:

A refueling is in progress when the audible count rate indication in the control room fails.

During the next hour, the following evolutions were expected to take place:

  • Move the secondary source from one core location to another
  • Remove 4 fuel assemblies from the core
  • Reposition the shoehorn
  • Reposition an underwater camera What is your direction as the Core Loading supervisor regarding the evolutions listed above?

A. Allow the movement of the secondary source, underwater camera, and shoehorn to occur, but the boron concentration must be verified within 12 hours1.388889e-4 days <br />0.00333 hours <br />1.984127e-5 weeks <br />4.566e-6 months <br />.

B. Allow all of the evolutions to occur as long as the boron concentration is determined immediately.

C. Do NOT allow the movement of the secondary source or fuel assemblies until the audible count rate indication in the control room is again operable.

D. Do NOT allow any of the evolutions to occur until the audible count rate indication in the control room is again operable.

Proposed Answer: C Explanation (Optional):

0POP08-FH-0009, Core Refueling, Section 5.5 identifies conditions where the Core Load Supervisor should suspend Core Alterations and includes if either Core Monitoring NI becomes INOP. Thus as given in the stem, the candidate should recognize that Core Alterations should be suspended.

0POP03-ZG-0010, Refueling Operations, is the administrative procedure for these activities and Core Alterations are defined in Sections 4.12 and 4.13. Per this procedure, moving the secondary source from one core location to another and remove fuel assemblies from the core are considered Core Alterations, and therefore are not allowed to be conducted until the audible count rate indication in the control room is again operable. Thus, C is the only correct answer.

Technical Reference(s): 0POP03-ZG-0010, Refueling Operations (Rev 39, Sections 4.12, 4.13, and 5.15);

0POP08-FH-0009, Core Refueling (Rev 27, Section 5.5)

Proposed references to be provided to applicants during examination: None Learning Objective: _________________________ (As available)

Question Source: Bank # _______

Modified Bank # _______ (Note changes or attach parent)

New ___X___

Question History: Last NRC Exam N/A Question Cognitive Level: Memory or Fundamental Knowledge _____

Comprehension or Analysis __X__

10 CFR Part 55 Content: 55.41 ____

55.43 _7__

Comments:

Original KA (G2.2.13 - Knowledge of tagging and clearance program) was REJECTED due to an SRO only question could not be constructed from this KA.

KA: Equipment Control, G2.2.26 - Knowledge of refueling administrative requirements.

ES-401, Rev. 9 STP SRO Written Examination Form ES-401-5 Question Worksheet Examination Outline Cross-

Reference:

Level SRO Tier # 3 Group #

K/A # G2.2.23 Importance Rating 3.8 Proposed Question 97:

Unit 1 is in Mode 1. The following series of events occur:

  • November 11, 0800: Only one offsite transmission network is available to the onsite Class 1E distribution system due to a fault in the Switchyard.
  • November 14, 0600: ESF DG C is declared INOPERABLE Which ONE of the following correctly identifies the required Tech Spec actions?

A. Be in Hot Shutdown by November 14, 2000, be in Cold Shutdown by November 15, 2000.

B. Be in Hot Shutdown by November 14, 2359, be in Cold Shutdown by November 16, 0600.

C. Be in Hot Standby by November 14, 2359, be in Cold Shutdown by November 16, 0600.

D. Be in Hot Standby by November 14, 1800, be in Cold Shutdown by November 15, 1800.

Proposed Answer: __A__

Explanation (Optional):

Only one offsite transmission network puts unit in LCO 3.8.1.1.a, (applicable in Modes 1-4) and Action a is entered at 0800, Nov 11th. Action a time limits are to restore offsite circuit within 72 hours8.333333e-4 days <br />0.02 hours <br />1.190476e-4 weeks <br />2.7396e-5 months <br /> or be at least in HOT SHUTDOWN within next 12 hours1.388889e-4 days <br />0.00333 hours <br />1.984127e-5 weeks <br />4.566e-6 months <br /> and in COLD SHUTDOWN within the following 24 hrs.

ESF DG A going INOP on Nov 12th puts unit in LCO 3.8.1.1.a and 3.8.1.1.c, and Action c is entered at 1600. Since ESF DG A is declared OPERABLE before the 12 hour1.388889e-4 days <br />0.00333 hours <br />1.984127e-5 weeks <br />4.566e-6 months <br /> time limit, Action c is exited at 0200 on Nov 13th. (Action a is still in effect with its original start time).

When ESF DG C is declared INOP, Action c is again entered at 0600 on Nov 14. Action c time limits are to restore at least one of the inoperable sources to OPERABLE status within 12 hours1.388889e-4 days <br />0.00333 hours <br />1.984127e-5 weeks <br />4.566e-6 months <br /> or be in at least HOT STANDBY within the next 6 hours6.944444e-5 days <br />0.00167 hours <br />9.920635e-6 weeks <br />2.283e-6 months <br /> and in COLD SHUTDOWN within the following 30 hours3.472222e-4 days <br />0.00833 hours <br />4.960317e-5 weeks <br />1.1415e-5 months <br />.

Action a time limit is to be in at least HOT SHUTDOWN by 2000 on Nov 14, and Action c time limit is to be in at least HOT STANDBY by midnight on Nov 14, therefore Action a is most limiting and must be complied with.

A: Correct. Action a is most limiting B: Incorrect. This distractor is calculated using Action c time requirements, but lists HOT SHUTDOWN, not HOT STANDBY as identified in the TS.

C: Incorrect. This distractor is calculated using Action c time requirements.

D: Incorrect. This distractor is calculated using Action a time requirements, but is applied to the date and time ESF DGC was declared INOP.

Technical Reference(s): TS 3.8.1, A.C. Sources Proposed references to be provided to applicants during examination: TS 3.8.1 Learning Objective: _________________________ (As available)

Question Source: Bank # _______

Modified Bank # _______ (Note changes or attach parent)

New ___X____

Question History: Last NRC Exam N/A Question Cognitive Level: Memory or Fundamental Knowledge _____

Comprehension or Analysis __X__

10 CFR Part 55 Content: 55.41 _____

55.43 __2__

Comments:

KA: Equipment Control, 2.2.23 - Ability to track limiting conditions for operations.

ES-401, Rev. 9 STP SRO Written Examination Form ES-401-5 Question Worksheet Examination Outline Cross-

Reference:

Level SRO Tier # 3 Group #

K/A # G2.3.1 Importance Rating 3.0 Proposed Question 98:

10CFR20, Standards for Protection Against Radiation, generally defines a High Radiation area as one in which an individual could receive a dose in excess of 0.1 Rem in 1 hour1.157407e-5 days <br />2.777778e-4 hours <br />1.653439e-6 weeks <br />3.805e-7 months <br />. Certain controls are mandated for these areas. At STP, what are the MINIMUM controls required by the Tech Specs for an area in which an individual could receive 0.5 Rem in 1 hour1.157407e-5 days <br />2.777778e-4 hours <br />1.653439e-6 weeks <br />3.805e-7 months <br /> in addition to an access RWP?

A. Barricaded and conspicuously posted as a High Radiation Area.

B. Barricaded and conspicuously posted as a High Radiation Area and have a control device that, upon entry into the area, causes the level of radiation to be reduced below a level in which an individual could receive 0.1 Rem in 1 hour1.157407e-5 days <br />2.777778e-4 hours <br />1.653439e-6 weeks <br />3.805e-7 months <br />.

C. Barricaded and conspicuously posted as a High Radiation Area and have a control device that energizes a conspicuous visible or audible alarm upon entry.

D. Barricaded and conspicuously posted as a High Radiation Area and is locked except for periods of access.

Proposed Answer: A Explanation (Optional):

Tech Spec 6.12.1 provides requirements for areas with radiation fields between 100 mr/hr and 1000 mr/hr. This range encompasses the 500 mr/hr field cited in the question.

The word MINIMUM was used in the question to establish the requirements cited in this TS section as those that apply as a minimum, thus anything in addition would be an incorrect answer. The distracters all use the TS info, but also include other controls cited in 10CFR20.

These other controls arent used at STP and this fact provides the basis of this TS as the first sentences indicates STP takes exception to the 10CFR20 controls, but provide the specified alternate means of control.

Technical Reference(s): Tech Spec 6.12.1 Proposed references to be provided to applicants during examination: None Learning Objective: _________________________ (As available)

Question Source: Bank # _______

Modified Bank # _______ (Note changes or attach parent)

New ___X___

Question History: Last NRC Exam N/A Question Cognitive Level: Memory or Fundamental Knowledge __X__

Comprehension or Analysis _____

10 CFR Part 55 Content: 55.41 ____

55.43 _5__

Comments:

KA: Radiation Control, G2.3.1 - Knowledge of 10CFR20 and related facility radiation control requirements.

ES-401, Rev. 9 STP SRO Written Examination Form ES-401-5 Question Worksheet Examination Outline Cross-

Reference:

Level SRO Tier # 3 Group #

K/A # 2.3.4 Importance Rating 3.1 Proposed Question 99:

Given the following:

  • Unit 1 is under a Site Area Emergency.
  • An individual has life-threatening injuries in an area where the radiation level is 70 Rem/Hr.
  • It will take 15 minutes in this area to attend to the individuals injuries and transport him out of the area.

In accordance with Emergency Plan Procedure 0ERP01-ZV-IN06, Radiological Exposure Guidelines, who can authorize the exposure limit for assistance personnel and which range of dose should be authorized?

A. Emergency Director, 5-10 Rem B. Emergency Director, 10-25 Rem C. Radiological Director, 5-10 Rem D. Radiological Director, 10-25 Rem Proposed Answer: __B__

Explanation (Optional): In this situation, the referenced procedure allows the Shift Manager to authorize any dose.

Technical Reference(s): OERP01-ZV-IN06, Radiological Exposure Guidelines (Rev 5, Addendum 1)

Proposed references to be provided to applicants during examination: None

Learning Objective: _________________________ (As available)

Question Source: Bank # _______

Modified Bank # _______ (Note changes or attach parent)

New ___X___

Question History: Last NRC Exam N/A Question Cognitive Level: Memory or Fundamental Knowledge __X__

Comprehension or Analysis _____

10 CFR Part 55 Content: 55.41 ____

55.43 _4__

Comments:

Original KA (G2.3.5 - Knowledge of use and function of personnel monitoring equipment) was REJECTED due to an SRO only question could not be constructed from this KA.

KA: Radiation Control, G2.3.4 - Knowledge of radiation exposure limits and contamination control, including permissible levels in excess of those authorized.

ES-401, Rev. 9 STP SRO Written Examination Form ES-401-5 Question Worksheet Examination Outline Cross-

Reference:

Level SRO Tier # 3 Group #

K/A # 2.4.18 Importance Rating 3.6 Proposed Question 100:

Given the following:

  • A reactor trip occurred coincident with a loss of offsite power.
  • 0POP05-EO-ES03, Natural Circulation Cooldown with Steam Void in Vessel, is in progress.
  • Steps 1 through 4 initiate RCS cooldown and depressurization.
  • Step 5 states CHECK RVWL indication - GREATER THAN OR EQUAL TO 85%

Which ONE of the following is the basis for verifying RVWL Plenum indication is at least 85%?

A. Ensures thermal stresses to the vessel flange are minimized.

B. Ensures RCS total mass does not drop below minimum conditions assumed in FSAR analysis for natual circulation cooldown.

C. Ensures that the steam void in the Reactor Vessel does not enter the hot legs and disrupt natural circulation flow.

D. Ensures that steam collection in the RCP impeller is minimized prior to pump start attempts in subsequent procedure steps.

Proposed Answer: __C__

Explanation (Optional):

Lesson Plan for this EOP states the basis for verifying RVWL Plenum indication at least 85% is as follows: BASIS: Alerts the operators to monitor void growth to prevent any of the void from entering the hot legs which could disrupt natural circulation flow.

Technical Reference(s): 0POP05-EO-ES03, Natural Circulation Cooldown With Steam Void in Vessel; LOT504.26.LP, Lesson Plan for 0POP05-EO-ES03 (Rev 5, Section 2.9)

Proposed references to be provided to applicants during examination: None Learning Objective: _________________________ (As available)

Question Source: Bank # _______

Modified Bank # _______ (Note changes or attach parent)

New ___X___

Question History: Last NRC Exam N/A Question Cognitive Level: Memory or Fundamental Knowledge __X__

Comprehension or Analysis _____

10 CFR Part 55 Content: 55.41 ____

55.43 _5__

Comments:

KA: Emergency Procedures / Plans, G2.4.18 - Knowledge of the specific bases for EOPs.